Elawyers Elawyers
Washington| Change
Find Similar Cases by Filters
You can browse Case Laws by Courts, or by your need.
Find 49 similar cases
MARK W. NELSON vs FLORIDA ENGINEERS MANAGEMENT CORPORATION, 98-005321 (1998)
Division of Administrative Hearings, Florida Filed:Gainesville, Florida Dec. 07, 1998 Number: 98-005321 Latest Update: Jul. 09, 1999

The Issue Whether Petitioner is entitled to additional credit for his responses to question numbers 21 and 24 of the Principles and Practice of Engineering Examination administered in April 1998.

Findings Of Fact Petitioner took the April 24, 1998 professional engineering licensing examination with an emphasis in civil engineering. A score of 70 is required to pass the test. Petitioner obtained a score of 69. In order to achieve a score of 70, Petitioner needs a raw score of 48. Therefore, Petitioner is in need of at least one additional raw score point. Petitioner is challenging question numbers 21 and 24. They are both multiple-choice questions and worth one point each. Exhibit 10 contains a diagram for the candidate's use in answering question numbers 21 and 24. Question 21 requires the examinee to calculate the percentage of wooded land on the diagram. The diagram contains a rectangle labeled "woodlot," and within the rectangle are three non-contiguous areas marked with schematics of trees. The Petitioner reduced the percentage of wooded area to conform to the portion of the area labeled "woodlot" marked with schematics of trees. In regard to question number 21, the Petitioner asserts that as a matter of convention, by failing to put the trees everywhere in the wooded lot, one may assume that there are trees only where there is a schematic of the trees. The Petitioner's challenge was rejected on the basis that the scorer opined that it is standard practice that drawings are only partially filled with details, and the most reasonable interpretation of the site plan drawings is that the woodlot fills the entire area enclosed by the rectangle. John Howath, a professional engineer, testified regarding accepted conventions in engineering drawings. In Howath's opinion the drawing on the examination used inconsistent methodologies and was confusing regarding whether all of the area designated by the label or "call out" of woodlot was in fact wooded. Both the Petitioner and Mr. Howath referred to drawings in the Civil Engineering Reference Manual which showed areas on drawings totally covered with visual indications of a particular material or condition. Peter Sushinsky, a professional engineer, testified as an expert for the Respondent. Mr. Sushinsky acknowledged the Petitioner's exhibits; however, Mr. Sushinsky noted that these were only a few examples of drawings that are available. Mr. Sushinsky referenced construction drawings he had seen in his practice with partial "cross-hatching" just like the diagram on the examination. In sum, Mr. Sushinsky's experience was that diagram might be totally or partially "cross-hatched." In Mr. Sushinsky's opinion it was not a bad diagram, only subject to a different interpretation by a minor group. Question number 24 asked the candidate to calculate the weir peak discharge from the catchment area using the rational formula. The Petitioner asserts the question is misleading and should read, "What is the peak discharge from the watershed?" The Petitioner bases his assertion on the ground that the "rational formula" is used to compute discharge from a watershed not a weir, as mandated by the question. The scorer did not address the Petitioner's concerns. The scorer stated, "It is clear from the item statement that the weir equation is not to be used." However, the questions ask the candidate to compute the weir discharge. Jennifer Jacobs, a professor of engineering, testified regarding the rationale formula that it was used to calculate watershed discharge and not weir discharge. All experts agreed that the rational formula is not used to compute weir discharge. The experts all agree that the question was confusing because the rational formula is not used to calculate the discharge from a weir. The Respondent's expert justifies the answer deemed correct on the basis that if one uses the rational formula and computes the watershed discharge, one of the answers provided is close to the result. The Respondent's expert calculated the watershed discharge as 230.6 cubic feet per second (cfs). The answer deemed correct was 232 cfs. The expert stated the weir attenuates flow. If the weir attenuates flow one would expect an answer less than 230.6 cfs., not an answer equal to or greater than 230.6 cfs. The amount of attenuation is based upon the physical features of the impoundment area and the mouth of the weir. Weir Attenuation varies. The only answers smaller than 230.6 are 200 or 32. Is the 232 cfs. answer wrong because it does not allow for attenuation by the weir? How much did the weir attenuate the flow? Under these facts, the question is capricious. The Respondent argues that the Petitioner didn't follow instructions while acknowledging that the "correct" answer is not the answer to the question that was asked.

Recommendation Based upon the foregoing Findings of Fact and Conclusions of Law set forth herein, it is RECOMMENDED: That the Respondent enter a final order awarding Petitioner two raw points and a passing score on the Principles and Practice of Engineering Examination. DONE AND ENTERED this 20th day of May, 1999, in Tallahassee, Leon County, Florida. STEPHEN F. DEAN Administrative Law Judge Division of Administrative Hearings The DeSoto Building 1230 Apalachee Parkway Tallahassee, Florida 32399-3060 (850) 488-9675 SUNCOM 278-9675 Fax Filing (850) 921-6847 www.doah.state.fl.us Filed with the Clerk of the Division of Administrative Hearings this 20th day of May, 1999. COPIES FURNISHED: Mark W. Nelson 720 Northwest 31st Avenue Gainesville, Florida 32609 Natalie A. Lowe, Esquire Board of Professional Engineers 1208 Hays Street Tallahassee, Florida 32301 Dennis Barton, Executive Director Board of Professional Engineers 1208 Hays Street Tallahassee, Florida 32301 William Woodyard, General Counsel Department of Business and Professional Regulation 1940 North Monroe Street Tallahassee, Florida 32399-0792

Florida Laws (1) 120.57
# 1
CARLOS A. REDDING vs CRIMINAL JUSTICE STANDARDS AND TRAINING COMMISSION, 07-005068 (2007)
Division of Administrative Hearings, Florida Filed:Tallahassee, Florida Nov. 02, 2007 Number: 07-005068 Latest Update: May 14, 2008

The Issue Whether Petitioner's challenge to the State Officer's examination should be sustained.

Findings Of Fact Petitioner took the State Officers Certification Examination (SOCE) on August 29, 2007. This was Petitioner's third time taking the examination, which he did not pass. While it is clear that Petitioner did not pass, no evidence was presented indicating what score was achieved on the examination. Likewise, no evidence was presented regarding the value of the questions challenged in this proceeding. Therefore, it cannot be determined on this record whether awarding credit for or discarding the two challenged questions would result in a passing score. Question 1281/ required the applicant to demonstrate knowledge of the formula used for calculating the speed a car was traveling from skid marks. The scenario in the question provided enough information for the test taker to answer the question correctly. The proposed answers placed different factors from the scenario in the formula. The correct answer fitting the formula was answer choice "C". Petitioner answered "B". Petitioner challenged the question because the correct answer reflected a whole number and resulted from "rounding up," when the training materials provided instructed students not to "round up." The question did not ask the applicant for the exact number, but asked that they identify the answer with the correct formula components. Petitioner's answer did not include the appropriate formula components. The correctness of Petitioner's answer was in no way affected by his complaint about "rounding up." Indeed, all of the available answers were whole numbers. Question 128 is statistically valid. Eighty-two percent of all applicants who have answered this question have answered it correctly. The question has been answered by 3,606 students. Of that number, 2,960 students have answered the question correctly, while only 399 have chosen the answer selected by Petitioner. Question 150 required the applicant to determine what charges could be considered against a person going under or attempting to go under a crime-scene tape. The scenario in the question provided enough information for the test-taker to answer the question correctly. Given the facts presented in the scenario for question 150, the correct answer was "D". Petitioner answered "C". Petitioner's challenge to the question is based upon assumptions related to the scenario that were not presented in the examination, coupled with a misreading of the training materials. Moreover, of the 1,126 applicants who have answered question 150, 757 students have answered the question correctly. Only 353 applicants have chosen the answer selected by Petitioner. Petitioner has failed to show that either question 128 or question 150 was unclear, ambiguous or in any respect unfair or unreasonable. Neither has he established that he answered either question correctly.

Recommendation Upon consideration of the facts found and conclusions of law reached, it is RECOMMENDED: That the Florida Department of Law Enforcement enter a final order rejecting Petitioner's challenge to the scoring on questions 128 and 150 of the SOCE and dismiss the petition in this proceeding. DONE AND ENTERED this 12th day of February, 2008, in Tallahassee, Leon County, Florida. S LISA SHEARER NELSON Administrative Law Judge Division of Administrative Hearings The DeSoto Building 1230 Apalachee Parkway Tallahassee, Florida 32399-3060 (850) 488-9675 SUNCOM 278-9675 Fax Filing (850) 921-6847 www.doah.state.fl.us Filed with the Clerk of the Division of Administrative Hearings this 12th day of February, 2008.

Florida Laws (7) 119.07120.569120.57943.13943.1397943.17943.173 Florida Administrative Code (1) 28-106.201
# 2
JONATHAN A. BATISTA vs BOARD OF PROFESSIONAL ENGINEERS, 20-003075RX (2020)
Division of Administrative Hearings, Florida Filed:Tallahassee, Florida Jul. 10, 2020 Number: 20-003075RX Latest Update: Jun. 09, 2024

The Issue The issue in this case is whether Florida Administrative Code Rule 61G15-21.004(2) is an invalid exercise of delegated legislative authority.

Findings Of Fact The Parties Petitioner is an applicant for licensure as a professional engineer ("P.E.")2 in Florida, and currently works in the discipline of environmental engineering in Florida. His practice focuses primarily on water-related areas within that discipline. Petitioner is not currently licensed as a P.E. Respondent is a board within the Department of Business and Professional Regulation ("Department"). It is the state agency created pursuant to section 471.007, Florida Statutes, and charged with licensing professional engineers in Florida. Respondent is vested with the authority to adopt rules to implement chapter 471, regarding the regulation of the practice of engineering in Florida, as defined in section 471.005(7). Respondent adopted the Challenged Rule at issue in this proceeding. Statutory and Rule Background The engineering profession in Florida is regulated pursuant to chapter 471. A person may become licensed as a P.E. in Florida by applying for licensure, fulfilling specified educational and experience requirements, and 2 For purposes of this Final Order, the terms "professional engineer" or "P.E." will be used to refer to persons who are licensed engineers under chapter 471, Florida Statutes. either being endorsed for licensure as provided in sections 471.015(3) and (5), or passing the required licensure examinations. § 471.015, Fla. Stat. Pursuant to section 471.015, Respondent has adopted Florida Administrative Code Chapter 61G15-20, which codifies, in rule, the requirements for licensure as a P.E. in Florida. An applicant for licensure must be a graduate of a Board-approved engineering program; have the requisite number of years of engineering experience; and have passed the specified licensure exams. Fla. Admin. Code R. 61G15-20.0010. Section 455.217(1)(d) authorizes Respondent to adopt, by rule, the use of a national professional licensing examination that the Department has certified as meeting requirements of national examinations and generally accepted testing standards. To implement section 455.217(1), Respondent has adopted rule 61G15- 21.001, titled "Examination Designated; General Requirements." This rule requires that, unless an applicant qualifies for licensure by endorsement, he or she must pass the National Council of Examiners for Engineering and Surveyors ("NCEES") licensure exam. Part I of the NCEES exam is the Fundamentals of Engineering ("FE") exam, and part II of the NCEES exam is the Principles and Practice Exam ("PP") exam. Respondent has entered into a contract with NCEES to provide the FE and PP exams in Florida. A person must pass both the FE and PP exams to be licensed as a P.E. in Florida. § 471.015(1), Fla. Stat. The Challenged Rule states: "[t]he passing grade for Principles and Practice Exam is determined by the National Council of Examiners for Engineering and Surveying, where psychometric statistical methods are used to determine the level of performance that corresponds with minimal competence in the discipline." Fla. Admin. Code R. 61G15-21.004(2). The Challenged Rule is a subsection of rule 61G15-21.004, which is titled "Passing Grade." The Challenged Rule specifically and exclusively addresses the method for determining the passing grade on the PP exam. Sections 455.217(1)(d) and 471.013 are cited as the rulemaking authority for the Challenged Rule, and sections 455.217(1)(d) and 471.015(1) are cited as the law implemented by the Challenged Rule. The term "engineering," as used in section 471.005(7), includes the term "professional engineering," and defines the types of services and creative work that constitutes "engineering." An "engineer," as defined in section 471.005(5), includes the terms "professional engineer" and "licensed engineer," and means a person who is licensed to engage in the practice of engineering under chapter 471. By contrast, an "engineer intern," as defined in section 471.005(6), means a person who has graduated from a Board-approved engineering curriculum and has passed the FE exam. By definition, these are distinct terms. The term "engineer" is used to describe a person licensed as a P.E. under chapter 471, while the term "engineer intern" is used to described a person who may engage in the kinds of activities described within the term "engineering," as defined in section 471.005(7), but who is not licensed as a P.E. in Florida, and, therefore, is not authorized to hold himself or herself out as a licensed engineer in Florida. The Rule Challenge Petition The Rule Challenge Petition alleges four grounds under section 120.52(8) for invalidating the Challenged Rule. Alleged Invalidity of Challenged Rule under Section 120.52(8)(c) In paragraph 15 of the Rule Challenge Petition, Petitioner asserts that section 455.217(1)(c)—which, at the time the Rule Challenge Petition was filed, was cited as the specific authority for, and law implemented by, the Challenged Rule—did not authorize the Challenged Rule, so that the Challenged Rule enlarged, modified, or contravened the specific provisions of law implemented, pursuant to section 120.52(8)(c). As a result of Petitioner having filed the Rule Challenge Petition, Respondent discovered that it had not updated its citation of the specific authority for, and law implemented by, the Challenged Rule, when section 455.217(1) was amended and renumbered in 1997, so that section 455.217(1)(c) no longer was the correct citation to the law implemented by the Challenged Rule. Respondent requested the Department of State, Administrative Code and Register Section ("DOS"), to make a technical, non-substantive change to the Challenged Rule. As authorized by Florida Administrative Code Rule 1-1.010(10), DOS updated the statutory citation to section 471.217(1)(d), which is the correct citation to the law implemented by the Challenged Rule. This technical change nullifies the alleged invalidity ground set forth in paragraph 15 of the Rule Challenge Petition, and Petitioner concedes this. In paragraph 16 of the Rule Challenge Petition, Petitioner also alleges that the Challenged Rule enlarges, modifies, or contravenes section 455.217(1)(a), because the PP examination does not adequately and reliably measure an applicant's ability to practice the profession regulated by the Department. However, as discussed below, section 455.217(1)(a) is not cited as a specific provision of law implemented by the Challenged Rule, so cannot form the basis of a challenge to the Rule under section 120.52(8)(c). Alleged Invalidity of Challenged Rule under Section 120.52(8)(d) In paragraph 17 of the Rule Challenge Petition, Petitioner alleges that the Challenged Rule is invalid under section 120.52(8)(d) because it is vague, fails to establish adequate standards for agency decisions, or vests unbridled discretion in the agency. In support of this alleged invalidity ground, Petitioner asserts that the Challenged Rule is vague because "the level of performance on the PP exam is stated to correspond with minimal competency, yet there are no rules which provide definitive guidance to NCEES on what constitutes the general areas of competency in regards to engineering practice."3 Petitioner also alleges, in paragraph 19 of the Rule Challenge Petition, that the Challenged Rule is invalid under section 120.52(8)(d) because it equates passage of the PP exam with a 30-year practice experience requirement for licensure by endorsement set forth in section 471.015(5)(b). To this point, Petitioner states: "I can't think of anything more arbitrary than the principles and practice exam equating to near[-]retirement level experience."4 Alleged Invalidity of Challenged Rule under Section 120.52(8)(e) In paragraph 18 of the Rule Challenge Petition, Petitioner alleges that the Challenged Rule is invalid pursuant to section 120.52(8)(e) because it is arbitrary, for several reasons. Paraphrased, these reasons are: passage of the PP exam does not accurately reflect, or equate to, minimal competence in the discipline; the PP Exam does not accurately evaluate an individual's engineering ability level, but instead evaluates an individual's exam performance compared to average group exam performance; the PP exam does not reliably distinguish between minimal competence and incompetence to practice engineering, as evidenced by the fact that engineers who fail the PP exam still competently perform, and, thus, keep, their engineering jobs; passing the PP exam, by itself, does not certify an individual to competently perform any engineering service or creative work as defined in 3 Section 455.217(1)(b) requires, for each exam developed by the Department or a contracted vendor, that the general areas of competency covered by the exam be specified by rule. The last sentence of that subsection states that the requirements of subsection (b) do not apply to national exams, such as the NCEES PP exam, which are approved and administered pursuant to section 455.217(1)(d). Thus, the law implemented by the Challenged Rule does not require areas of competency to be specified in the Challenged Rule. 4 Because paragraph 19 of the Rule Challenge Petition alleges that the rule is arbitrary, the undersigned considers this paragraph to constitute a challenge to the Challenged Rule under section 120.52(8)(e), rather than under section 120.52(8)(d), as cited in the Rule Challenge Petition, and has addressed this ground in the Conclusions of Law section dealing with that alleged invalidity ground. section 471.005(7); the PP exam does not reliably determine if an examinee is minimally competent, due to an incorrect reference point; and the PP exam does not reliably distinguish between individuals whose practice of engineering would protect the public health and safety and those whose practice of engineering would constitute a danger to public health and safety. Alleged Invalidity of Challenged Rule under Section 120.52(8)(f) In paragraph 20 of the Rule Challenge Petition, Petitioner alleges that the Challenged Rule is invalid pursuant to section 120.52(8)(f) because it imposes regulatory costs on the regulated person, county, or city, which could be reduced by the adoption of less costly alternatives that substantially accomplish the statutory objectives. To this point, Petitioner proposes a state- conducted investigation of an applicant as a substitute for the PP exam, and contends that "there's a premium associated with a national exam. It's also clear that the regulatory cost imposed on me and all future examinees could be substantially reduced if the Board conducted their exam as an investigation and did not incorporate the NCEES exam." However, as more fully discussed below, this challenge ground is time- barred by section 120.541(1)(g), and, therefore, is not a legally-cognizable basis for invalidating the Challenged Rule in this proceeding. The Parties' Stipulated Facts Petitioner is an Engineer Extern, Texas EIT 56990. Pursuant to section 471.015(1), the Florida Engineers Management Corporation "shall issue a license to any applicant who the Board certifies is qualified to practice engineering and who has passed the [FE] exam and the [PP] exam." Both the FE exam and the PP exam are created by NCEES, pursuant to section 455.217(1)(d), which states, in pertinent part: "a board . . . may approve by rule the use of any national examination which the [Department of Business and Professional Regulation] has certified as meeting the requirements of national examinations and generally accepted testing standards pursuant to department rules." Pursuant to section 471.015(1), Petitioner passed the FE exam on September 3, 2016. Passing the FE is a prerequisite to take the PP exam. Petitioner registered to take the NCEES computer-based PP exam for environmental engineering on April 22, 2020. He paid the $350 exam fee and additional monies for test preparation material, and spent at least 100 hours preparing for the exam. He was notified, on April 6, 2020, by Pearson Vue, the test center company, that the exam was cancelled due to the Covid-19 pandemic. On April 7, 2020, Petitioner registered to take the exam on July 15, 2020, which was the earliest available date for taking the exam in his local area. On April 9, 2020, Petitioner canceled his registration for the July 15, 2020, exam and decided to apply for licensure as a P.E. without passing the PP exam. Pursuant to section 471.015(2)(a)1., on May 18, 2020, Petitioner filed his application for licensure with the Board. The application provides information stating that Petitioner meets the requirements of section 471.013(1)(a)1., and has at least four years of active engineering experience of a character indicating competence to be in responsible charge. In compliance with the education requirements of section 471.013(1)(a)1., Petitioner earned a Bachelor of Science degree in Chemical Engineering from the University of Florida, which is accredited by ABET. Petitioner provided an official transcript to the Board for verification purposes. In compliance with rule 61G15-20.002, Petitioner states in his application that he has at least four years of acceptable engineering practice, consisting of one year of experience equivalent through his Master of Engineering degree from the University of Texas at Austin, an ABET- accredited institution; and over three years of professional engineering work verified by licensed engineers. According to rule 61G15-20.002, an applicant must list three current personal references who are professional engineers. Thirteen licensed engineers submitted documentation to the Board regarding Petitioner's type of qualifying experience, level of engineering competency, and professional integrity. All references circled "yes" to the question "would you employ the applicant in a position of trust?" Two additional references in the engineering industry also provided their opinion on Petitioner's integrity and competence. On June 1, 2020, Petitioner received notification from the Board that his Florida 0901 1031-P.E. Endorsement application was incomplete. Petitioner addressed all items listed in a timely fashion to participate in the June 10, 2020, Board meeting. The Board stated during an informal hearing on June 10, 2020, that Petitioner would not be granted P.E. licensure due to not having passed the PP Exam. Petitioner is substantially affected by the Challenged Rule because it disqualifies him from becoming a licensed engineer in Florida without passing the PP exam. Findings of Fact Based on Evidence Presented at the Final Hearing A. Findings Regarding the Evidence Presented in Petitioner's Case Testimony of Witnesses Hoot and Grace David Hoot and Nigel Grace, both of whom are licensed professional engineers, testified regarding Petitioner's abilities and skills as a practicing engineer. Neither Hoot nor Grace was qualified as an expert in the field of psychometrics or related topics. Therefore, any opinions regarding these subject matters to which they testified at the hearing have been treated as personal opinions, rather than expert opinions. Hoot characterized Petitioner as a good, diligent young engineer, and described Petitioner's role in various projects on which they worked together. Based on his work with Petitioner, it was Hoot's personal opinion that Petitioner possesses the integrity and competence to work as a licensed engineer who would serve the public health and safety. Hoot testified that when he took the PP exam approximately 38 years ago, it was a free response exam. He stated that he was "not exactly sure" that a multiple-choice exam captures an examinee's ability to apply reason and judgment, but he acknowledged that he does not know how the PP exam currently is developed. He offered his personal opinion that it was possible for a competent engineer to fail the PP exam. Hoot also offered his personal view that engineering licensure constitutes a standard of care; however, he did not think anything provides a guarantee of competence. It was his personal view that experience tended to make one more competent. He also offered his personal view that as an engineer gains more experience and becomes more specialized, it is understandable that he or she would not score as well as an engineer who is gearing up toward the four-year experience goal which enables them to apply for licensure as a P.E. As Hoot put it, "life happens. You have children involved. You move jobs. I think you . . . maybe have less time to study. . . you get farther away from the study habits of . . . learning to be able to take tests." Grace, who is employed as a P.E. with Brown and Caldwell ("B&C"), a large U.S. engineering firm, testified regarding Petitioner's work experience while he was employed by B&C. Petitioner's experience included working on drinking water projects, utilities, upgrading process equipment, site evaluations, bench scale testing, and other projects. Based on Grace's work with Petitioner, it was his personal opinion that Petitioner possesses the integrity and competence to work as a licensed engineer who would serve the public health and safety. Grace took the PP exam approximately 28 years ago, and at the time, a major portion of the exam consisted of long-form written exam questions that provided the opportunity for examinees to provide free response answers and earn partial credit for partially-correct answers. He testified that the exam also had a multiple-choice component. Grace testified that, "based on instinct," he knows well-designed multiple-choice questions can provide the same insight into an examinee's decision-making judgment as long-form questions. Grace's personal view is that passing the PP exam does not guarantee competence, and it is possible for an engineer to be competent in some engineering disciplines but not others. Grace also agreed that it was logical that an examinee with approximately four years' experience would perform better on the PP exam because he or she would have better-honed test-taking skills and be fresher in some areas tested on the exam. By contrast, engineers who have practiced longer have more experience, but often have become specialized and further removed from the test-taking environment. Thus, passing a broad-spectrum exam becomes a bigger hurdle for engineers who have practiced longer. Petitioner's Testimony Petitioner testified on his own behalf at the final hearing. As discussed above, Petitioner holds a bachelor of science degree in chemical engineering from an ABET-accredited institution, and holds a master of engineering degree from an ABET-accredited institution. Although Petitioner, through his training as an engineer, is skilled at mathematics and statistics, he is not trained, and does not have any substantial experience, in the field of psychometrics.5 5 As discussed in greater detail below, psychometrics is a specialized field of study that concerns the theory and technique of objective psychological measurement of skills, knowledge, abilities, and educational achievement. Petitioner acknowledged that he does not have a degree in psychometrics and that he is not trained in, or knowledgeable about, preparing and administering high-stakes professional examinations. At the time he filed the Rule Challenge Petition, Petitioner had no personal knowledge of the work done by psychometricians; did not know what a "cut score" was; and was not familiar with the Modified Angoff Method, item response theory, the specifics of converting raw scores to scaled scores, or other any psychometric tools and methods employed to prepare and score the PP exam. Petitioner acquired some rudimentary knowledge in a few of these areas in preparing for the final hearing in this proceeding.6 Section 90.701(2), Florida Statutes, prohibits a lay witness from testifying as to an opinion regarding a matter involving specialized knowledge.7 Accordingly, it is determined that Petitioner is not competent to provide an expert opinion regarding psychometrics and related areas, such as item response theory. He was not qualified, tendered, or accepted at the final hearing as an expert in psychometrics or any related areas. Because Petitioner was not qualified to testify as an expert witness at the final hearing, Petitioner's testimony regarding psychometrics, item response theory, scaled scores, the Modified Angoff Method, high-stakes professional testing, and all other specialized subject matters, consisted of opinion testimony by a lay witness. The only instances in which a lay witness 6 Petitioner does not have any special knowledge, formal training, education, or experience in the specialized field of psychometrics. His knowledge about these areas was acquired by reading and study in preparation for the final hearing. Because Petitioner lacks special knowledge, experience, training, and education in psychometrics, he is not qualified to testify as an expert in psychometrics or related topics, such as item response theory. See Chavez v. State, 12 So. 3d 199, 205 (Fla 2009)(in determining whether a witness is qualified to render an opinion as an expert in a specialized field, the court must determine whether the witness is adequately qualified to render an opinion based on special knowledge, experience, training, or education). 7 Chesser v. State, 30 So. 3d 625 (Fla. 1st DCA 2010)(it is error for a court to accept opinion testimony of a lay witness in a specialized subject matter area). may provide opinion testimony are when the lay witness's opinion is based on firsthand knowledge through personal perception.8 As the undersigned explained during the final hearing, rather than excluding Petitioner's lay opinion testimony, she would consider these pertinent evidentiary principles in determining the weight to be afforded Petitioner's lay opinion testimony in this proceeding. Applying these evidentiary principles, it is determined that Petitioner's testimony regarding psychometrics and related topics, such as item response theory, classical response theory, and high-stakes test reliability and validity, concerned specialized subject matters not within the realm of common knowledge or based on Petitioner's personal perception. Rather, such specialized subject matters required expert witness testimony, pursuant to section 90.702, and as addressed above, Petitioner was not shown to be an expert in any of these specialized subject matters. Because Petitioner's testimony constituted the type of opinion testimony that is not permissible by a lay witness, pursuant to section 90.701, such testimony is not afforded weight in this proceeding. Petitioner testified that "minimal competence," which is the standard measured on the PP Exam, equates to "competence" as defined in the dictionary—that is, the "sufficient ability for a specific need or requirement." However, this position ignores that, for purposes of the Challenged Rule, "minimal competence" is a term of art specifically used, in the psychometric measurement context, to describe the level of competence that corresponds to a passing score on the PP exam. As more fully discussed below, the PP exam is developed and scored using psychometric tools and methods. 8 Nat'l Commc'ns. Indus., Inc. v. Tarlini, 367 So. 2d 670, 671 (Fla. 1st DCA 1979)(lay witness testimony regarding a specialized subject matter was not admissible into evidence because the testimony was not regarding a subject matter about which the witness could testify based on common knowledge or his personal perception.) Petitioner contends that engineering experience is, by itself, a reliable measure of competence, so it is unnecessary to also pass the PP exam. Petitioner testified "the application process and [PP] exam have two entirely different methods to identify the same result: whether an engineer in training is competent enough to become a licensed engineer It is not logical for two checks of competence to come up with different results. There should be consistency." In support of this position, Petitioner relies on section 471.015(2)(a), which requires "at least 4 years of active engineering experience of a character indicating competence to be in 'responsible charge' of engineering." § 471.015(2)(a), Fla. Stat. "Responsible charge" is defined in rule 61G15-18.011(1) as the degree of control an engineer is required to maintain over engineering decisions made personally or by others over which the engineer exercises supervisory direction and control authority. An engineer in responsible charge is the "engineer of record," as defined in rule 61G15-30.002(1). Rule 61G15-30.002(1) defines "engineer of record" as a Florida professional engineer who is in responsible charge. Thus, an engineer who is qualified, for purposes of being in responsible charge pursuant to section 471.015(2)(a) must, in addition to having the minimum statutory experience, be a licensed P.E. This means that he or she necessarily must have passed the PP examination. These statutory and rule provisions collectively reinforce the point that for an engineer to demonstrate competence for purposes of holding himself or herself out as an "engineer," as defined in section 471.005(7), he or she must satisfy all three requirements of section 471.015(2)(a)—i.e., education, experience, and passing the licensing exam.9 9 The requirement to meet these three requirements, including the PP exam, is codified in section 471.051(2)(a). Eliminating the exam requirement and relying strictly on education and/or experience for licensure would require the Florida Legislature to amend this statute Petitioner echoed the testimony of Hoot and Grace that licensure is not a guarantee of competence, and that passing the PP exam does not guarantee minimal competence. To this point, he testified that he does not believe that the PP exam adequately and reliably measures an applicant's ability to practice engineering, and that experience is a better indicator of competence than passing the exam. By way of example, Petitioner described his own experience10—which he characterized as "directly matching" the activities in which a licensed engineer engages—and compared that experience to measuring competence by an exam, which Petitioner characterized as "attempt[ing] to indirectly measure my ability as an engineer." Based on his personal experience, Petitioner contends that experience better demonstrates competence to be licensed as a P.E.; that passing the PP exam does not indicate minimal competence to practice engineering; and that failing the PP exam does not mean that the examinee is not minimally competent. He further testified that examinees who fail the PP exam likely are minimally competent, since the engineering jobs they hold when applying for licensure likely would require that they be minimally competent in order to have been hired.11 to eliminate the exam requirement. The undersigned is not authorized by statute or the Florida Constitution to eliminate the PP exam requirement for licensure under chapter 471. 10 Petitioner's experience, set forth in his P.E. licensure application, was verified by his supervising engineers. 11 Petitioner appears to conflate being determined not "minimally competent" for purposes of passing the PP exam, with "incompetence," which is defined in Florida Administrative Code Rule 61G15-19.001(5) as the "physical or mental incapacity or inability of a professional engineer to perform the duties normally required of the professional engineer." Part of this confusion may be due, in part, to Respondent's response to one of Petitioner's interrogatories asking for a definition of "minimal competence." Rather than directly answering the interrogatory, Respondent referred Petitioner to the definition of "incompetence" for purposes of imposing discipline under Respondent's disciplinary rules—thus causing Petitioner to understandably assume that failing to demonstrate minimal competence through passing the PP exam equates to "incompetence," as defined in rule 61G15-19.001(5). However, the fact that Petitioner has not demonstrated "minimal competence" on the PP exam does not mean that he is incompetent; it simply means that he has not yet passed the PP exam for licensure as a P.E. in Florida. To this point, if failing to demonstrate "minimal competence" by passing the PP exam equated to being incompetence, every person who performs engineering work in To further illustrate this point, Petitioner noted that the data regarding passage rate of the PP exam shows that examinees having zero years of experience are almost twice as likely to pass the PP exam as examinees having 11 or more years of experience. However, as Hood and Grace explained, and as further discussed below, engineers having more than four to five years of experience begin to specialize in narrower fields and "life happens," in that personal and professional circumstances render it more difficult to prepare for and take a high-stakes test. Petitioner also disputed the accuracy of the PP exam preparation and scoring process. In particular, he took issue with the "model law engineer" standard to which the exam is designed. As discussed more extensively below, this standard equates to the competence level of an engineer having four years of engineering experience and who is capable of practicing engineering in a manner that protects the public health and safety. In particular, Petitioner contends that designing the PP exam to the "model law engineer" standard is unfair to anyone taking the exam that does not have exactly four of years of engineering experience. Notably, however, section 471.015(1), which is the statute implemented by the Challenged Rule, establishes four years as the engineering experience required for licensure as a P.E. Thus, the "model law engineer" standard is rationally related to the statutory minimum experience level for purposes of demonstrating minimum competency to be licensed. Petitioner also contends that the subject matters tested on the PP exam are unfairly broad, so that engineers who specialize in a particular area within an engineering discipline—such as specializing in water-related areas in environmental engineering—are disadvantaged by being required to take Florida but has not passed the PP exam would be "incompetent," and, thus, potentially subject to disciplinary action. an exam that covers a broad range of areas beyond his or her area of specialty. Petitioner further contends that it is irrational to test an examinee on particular areas that are irrelevant to his or her work and/or desired career path. However, the PP exam for a particular discipline is specifically designed to ensure that a licensed P.E. is competent to practice over a range of specific areas encompassed within that particular discipline. This is because once a person becomes a licensed P.E., he or she may practice engineering within any discipline or specific area within that particular discipline, subject to the professional and ethical requirements to limit practice to the disciplines and areas in which the engineer is actually competent. Thus, the breadth of the PP exam is designed to help ensure minimal competence to practice engineering in a manner that protects the public health and safety. Petitioner also contends that because the PP exam for some engineering disciplines tests a broader range of areas than the PP exam may test for other disciplines, the exam inconsistently measures minimal competency across the range of engineering disciplines. However, as discussed in detail below, the subject matters tested on the PP exam for a given discipline are chosen by subject-matter experts who are licensed engineers practicing in that particular engineering discipline, and are deemed, by those subject matter experts, to be most important to test for purposes of measuring competency in that discipline. Thus, while the number of discrete subject matters tested on the PP exams may differ across the various engineering disciplines, this difference is, factually and logically, a function of expert consensus regarding which subject matters need to tested to demonstrate minimal competence. Petitioner also contends that the Challenged Rule is vague because it does not specifically identify the disciplines, and the areas within each discipline, that are tested on the PP exam. Respondent has contracted with NCEES to be responsible for preparing, administering, and scoring the PP exams, pursuant to section 455.217(1)(d). NCEES conducts a methodical process, discussed in detail below, to determine the specific disciplines for which to develop a PP exam and the areas to be covered on the PP exam for a discipline. Exam specifications are then developed by subject matter experts within that discipline, and are published by NCEES. These specifications inform prospective examinees regarding the particular areas that will be tested on the PP exam for the discipline, and the number of questions for each specific area that will appear on the exam. Thus, prospective examinees are not left to wonder or guess about which disciplines will be tested; the areas within each discipline that will be tested; or the relative weight that will be assigned to each area tested.12 Although the Challenged Rule does not identify the specific disciplines tested on the PP exam, rule 61G15-21.001(1)(b)—which actually adopts the PP exam as an engineering licensure exam in Florida—states that the PP exam "is given by discipline." Therefore, even if section 455.217(1)(d) required the specific areas of competency to be identified by rule, such areas would have been identified in rule 61G15-21.001(1), rather than in the Challenged Rule.13 12 See Cole Vision v. Dep't of Bus. And Prof'l Reg., 688 So. 2d 404, 410 (Fla. 1st DCA 1997)(a rule is impermissibly vague if it is drafted in terms so vague that men of common intelligence must necessarily guess at its meaning or application. 13 Neither sections 455.217(1)(d) nor 471.015(1) specifically authorize or require Respondent to adopt rules identifying the general areas of competency tested on the PP exam. By contrast, exams developed by the agency pursuant to section 455.217(1)(b) must identify, by rule, the general areas of competency to be tested. Had the Legislature intended for exams authorized under section 455.217(1)(d)—of which the PP exam is an example—to adhere to the same requirement, the statute would have so stated. See Pro-Art Dental Lab, Inc. v. V- Strategic Grp., LLC, 986 So. 2d 1244, 1258 (Fla. 2008)(the specific mention of one thing in a statute implies the exclusion of another). Furthermore, section 120.54(1)(g) expressly requires a rule to address only one subject. Thus, if the Challenged Rule also addressed the areas of competency to be tested on the PP exam, it would violate section 120.54(1)(g). Petitioner also asserted, at the final hearing, that Respondent did not certify the PP exam as meeting the requirements of national examinations and generally accepted testing standards pursuant to department rules, as required by section 455.217(1)(d). However, Petitioner did not raise this alleged invalidity basis in the Rule Challenge Petition, so he is foreclosed from raising and litigating it at the final hearing. See § 120.56(1)(b), Fla. Stat. Petitioner also testified that, in transitioning from paper-and-pencil PP exams to computer-based exams, NCEES is relying on two different theories—classical test theory and item response theory—and that this reliance does not comport with generally accepted testing standards. However, as discussed above, Petitioner was not qualified as an expert in the specialized area of high-stakes examination preparation and scoring; thus, his testimony constitutes lay opinion regarding this specialized subject matter. He did not present any competent substantial evidence to support his contention that the PP exam does not meet generally accepted testing standards.14 Petitioner also testified that item response theory, which is a psychometric tool used in developing and scoring the PP exam, is an invalid means of determining the competence of an engineer. To this point, Petitioner testified that the "model law engineer" is not a real person, but is instead an imaginary person created by subject matter experts to define what a minimally competent engineer should know. Thus, according to Petitioner, 14 Because Petitioner was not qualified, tendered, or accepted as an expert in these specialized subject matters, and because his testimony on these matters consists of inadmissible lay opinion testimony, this testimony has not been afforded weight. §§ 90.701 and 90.702, Fla. Stat. the model law engineer standard is the wrong reference point for determining minimal competency to practice engineering.15 Petitioner also testified that the PP exam does not accurately measure ability, which is a latent trait for which an arbitrary measurement scale must be created. He testified that the model law engineer standard is the midpoint of this scale, and that the purpose of the scale is to determine whether examinees fall above or below that midpoint.16 He further contended that the PP exam does not accurately measure ability, because performance on the exam may be influenced by extraneous variables, such as test anxiety. Petitioner also testified regarding item response theory, which, as previously noted, is a psychometric tool used in developing and scoring high- stakes exams—a subject about which Petitioner had no training in, or knowledge about, until he prepared for the final hearing in this proceeding. Specifically, Petitioner testified that the item characteristic curve is the basic building block of item response theory, and that there are two technical properties of an item characteristic curve: difficulty of the item, and the ability of the item to discriminate between examinees' abilities. Petitioner testified that another basic principle of item response theory is that the examinee's ability is a variable with respect to the items used to determine it. According to Petitioner, this principle rests on two 15 Petitioner's testimony on this point was based on excerpts from a book titled Item Response Theory and a book titled The End of Average. The Item Response Theory book is a treatise on psychometrics, a highly specialized field about which Petitioner was not qualified to testify as an expert, and which is not susceptible to lay witness opinion testimony. Thus, Petitioner's testimony on these points is not assigned weight. See §§ 90.701 and 90.702, Fla. Stat. Additionally, excerpts from The End of Average were determined irrelevant, so were not admitted into evidence. 16 As support for this testimony, Petitioner selectively cited and quoted the deposition testimony of Timothy Miller, Respondent's expert on the development and scoring of NCEES's psychometric-based PP exams. The specific context of Miller's deposition testimony was that when an exam item is overexposed, it is subject to drift, which means that the percentage of correct answers for the item increases to the point that the item no longer is a good discriminator. As further discussed below, Petitioner's testimony on this point was directly and persuasively countered by Miller's expert testimony regarding scaled scores and setting the passing score for the PP exam. conditions: that all items measure the same underlying latent trait, and the values of all item parameters are in a common metric. According to Petitioner, this principle reflects that the item characteristic curve spans the entire ability scale; thus, the practical implication is that a test located anywhere along the ability scale can be used to estimate an examinee's ability, such that an examinee could take a test that is easy or hard, and on average, would score at the same estimated ability level. Petitioner testified that this stands in contrast to classical test theory, which he contends is a better discriminator of examinee ability.17 Petitioner also testified that the psychometric methods used to develop and score the computer-based PP exams are flawed because "difficulty" is subjective and entirely dependent on the individuals developing the PP exam. Thus, according to Petitioner, in scoring a computer-based multiple-choice PP exam, it is impossible to know whether a particular examinee got the answer right due to a reasonable approach in answering the question, or by guessing. Petitioner contends that for this reason, multiple-choice test questions developed using item response theory are not good discriminators of examinees' ability; thus, even if an examinee does not correctly answer enough questions to pass the exam, that does not mean that the examinee is not knowledgeable in that area.18 Petitioner further testified that because difficulty is a subjective parameter, different forms of the PP exam inherently have different levels of difficulty. Thus, according to Petitioner, it is a matter of luck whether an examinee takes a more difficult form or an easier form of the exam. Further to this point, Petitioner testified that because an examinee does not take multiple forms of the exam, but instead takes only one form, the determination of the examinee's ability is solely dependent on a subjective 17 Refer to note 15, supra. 18 Refer to note 15, supra. parameter—i.e., the difficulty of the test questions as determined by subject matter experts. Petitioner contends that, as a result, the PP exam does not accurately measure an examinee's ability, and, therefore, is not a valid exam.19 Petitioner also testified that because statistical indices of reliability and validity are not attributes of an exam, a researcher may select what seems to be an appropriate test for his or her purposes, when, in fact, the selected test does not have any level of reliability or validity. Thus, Petitioner testified, reliability and validity are values that reside in test scoring, not in the test itself. Petitioner testified that validation, in statistics, is the process of accumulating evidence that supports the appropriateness of the inferences that are made of student responses for assessment uses. He testified that validity refers to the degree to which the evidence indicates these interpretations are correct and the manner in which the interpretations are used is appropriate.20 Petitioner testified regarding three types of validity evidence: content, construct, and criterion evidence.21 Specifically, Petitioner testified that content evidence refers to the extent to which an examinee's responses to a given assessment reflect the examinee's knowledge of the content being tested; thus, to the extent an exam inadvertently measures a parameter that is not related to the examinee's knowledge of the content being tested, it is invalid. 19 Refer to note 15, supra. 20 Petitioner's testimony relied on, or was paraphrased from, a document titled The Scoring Rubric Development. Again, because this topic and document address a matter within the specialized fields of psychometrics, high-stakes testing, and test-scoring statistics, which are areas in which Petitioner was not qualified as an expert, and which are not susceptible to lay opinion testimony, pursuant to sections 90.701 and 90.702, Petitioner's testimony relying on this document, including his testimony regarding content, construct, and criterion-related evidence, is not afforded weight. 21 Refer to note 15, supra. This determination regarding the weight afforded Petitioner's testimony applies to paragraphs 94 through 100 herein. According to Petitioner, the content-related evidence for the PP exam for each discipline is inconsistent, so that the PP exam for a given discipline does not accurately measure minimal competence for that discipline. Petitioner also testified that the weighting of different topics on the PP exam necessarily creates an advantage for engineers who work in areas more heavily weighted on the exam, while creating a disadvantage for engineers who work in areas that are less heavily weighted on the exam. Petitioner also testified that to accurately determine minimal competence in all engineers, the model law engineer standard should be keyed to, and the content tested on the exam should be directed toward all engineers, including those having more than four years of experience. Petitioner also testified that construct-related evidence consists of external benchmarks, such as results and explanations, of internal evidence of psychological processes, such as reasoning. Petitioner testified that because multiple-choice exams do not provide evidence of an individual's reasoning process, they do not generate construct-related evidence for purposes of determining exam validity. Petitioner testified that free response paper-and pencil-exams provide construct-related evidence, so are better indicators of an examinee's knowledge. Petitioner also testified regarding criterion-related evidence, which relates to the extent to which the results of an assessment, such as the PP exam, correlate with a current or future event. By way of illustration, Petitioner testified that criterion-related evidence considers the extent to which a student's performance may be generalized to other relevant areas. Petitioner testified that an examinee's performance on the PP exam is not generalizable to other relevant activities, so it is impossible to determine whether the exam actually corresponds to minimal competence in the workplace. In sum, Petitioner testified that the PP exam does not meet content, construct, or criterion-based evidence for purposes of determining whether it is a valid exam. Thus, Petitioner reasons, it is logical to conclude that because the PP exam is not a good discriminator between minimally competent and incompetent engineers, it does not reliably and adequately measure competence. Petitioner also testified that because passing the PP exam is only one component of licensure, it fails to meet criterion-based validity, in that the exam, by itself, does not certify a passing examinee to practice as a P.E. As Petitioner put it, "you're just passing the exam as part of the requirement for licensure." Petitioner reasoned that if passing the PP exam corresponds to minimal competence, the experience and education requirements of section 471.015(2)(a) are redundant. Also to this point, Petitioner testified that the preapproval process to take the PP exam is directly related to an examinee's actual work experience as an engineer, while taking the exam merely entails answering questions about engineering work. Thus, Petitioner contends, a competent engineer, as shown through Respondent's preapproval process, may nonetheless fail the exam. Petitioner asserts that this further shows that the exam does not accurately measure minimal competence. Petitioner also testified that, in his view, delaying licensure of potentially competent engineers due to postponing the exam due to the Covid-19 pandemic does not serve the interest of public health and safety. To that point, he testified that the inability to obtain a variance, which would relieve examinees from having to take and pass the exam under such circumstances, renders the Challenged Rule arbitrary.22 Petitioner also contended that passing the PP exam should not be required, because other engineering professional associations—specifically, the European Federation of National Engineering Associations ("FEANI"), which represents engineers in European countries—allow licensure through 22 Petitioner's point regarding inability to obtain a variance or waiver is addressed in the Conclusions of Law. education and experience requirement, without requiring a professional exam to be taken and passed. However, because section 471.015, which governs the licensure of engineers in the state of Florida, requires a professional licensure examination to be taken and passed as part of the P.E. licensure requirements, Petitioner's testimony and argument regarding FEANI's practices and requirements take issue with the statute, rather than the Challenged Rule, and, thus are irrelevant. Petitioner also contends that the examination fee for the PP exam is arbitrarily set, rendering the Challenged Rule arbitrary. However, as discussed above, the Challenged Rule only addresses determining the passing grade for the PP exam using psychometric methods. The Challenged Rule has nothing to do with establishing or setting an examination fee. Thus, this challenge ground has no basis in fact or law.23 C. Findings Regarding the Evidence Presented in Respondent's Case Respondent's Expert Witnesses Respondent presented the testimony of Timothy Miller, P.E., who serves as Director of Examination Services for NCEES. Miller has held this position for approximately 15 years. His job-related activities and responsibilities include directing exam development, publication, scoring, and fulfillment of the licensing exams for engineers and surveyors; coordinating exam development committees consisting of over 800 volunteer subject matter experts who work on developing each NCEES exam; overseeing the exam development process and providing advice and guidance regarding engineering exam development, administration, production, scoring, analysis, and reporting; serving as a testing process consultant to exam development committees; and other exam-development and administration-related matters. 23 Additionally, this challenge ground was not raised in the Rule Challenge Petition, so is not at issue in this proceeding. See § 120.56(1)(b), Fla. Stat. Before Miller was promoted to his current position, he served as an examination development engineer for NCEES. In that position, Miller was responsible for planning and coordinating engineering exam development, production, administration, scoring analysis, and reporting for certain assigned examinations; serving as a testing consultant working with engineering exam development committee chairs regarding quality and number of exam development volunteers; and overseeing development and administration of the licensing exams in the specific fields of environmental controls systems, metallurgical engineering, and mechanical engineering. Through his experience in these positions with NCEES, Miller is an expert in professional examination development and scoring, particularly with respect to the development and scoring of the NCEES FE and PP examinations. Before being employed with NCEES, Miller practiced civil and structural engineering with several private-sector engineering firms. He has been a professional engineer since 1984, and is licensed as a P.E. in South Carolina, North Carolina, Maryland, Delaware, Pennsylvania, and New Jersey. Respondent also presented the testimony of Dr. Michelle Rodenberry, P.E., an associate dean and associate professor at the Florida A&M University–Florida State University College of Engineering. Her engineering expertise is in the field of structural engineering—specifically, bridge engineering. Rodenberry was appointed to the Board in 2012, and she served as a Board member until 2018. She is now an emeritus Board member. While on the Board, she served as chair of the education committee, and was involved in reviewing applications for licensure as a P.E. in Florida. Development, Scoring, and Validation of the PP Exam The NCEES engineering exams are national licensing exams that are recognized by every engineering licensing entity in each of the U.S. states, as well as by the engineering licensing entities in Washington, D.C.; Puerto Rico; the U.S. Virgin Islands; and the other U.S. territories and protectorates. There are approximately 26 different engineering disciplines, each of which is tested by a separate PP exam specific to that discipline. In the 1990s, NCEES decided to transition from subjectively-graded pencil-and-paper examinations to an objectively-graded computer-based multiple-choice exam format. Currently, approximately one-third of the PP exams, including environmental engineering, have been converted to a computer-based format, and all but one of the exams in the other disciplines is in the process of being converted. The reason NCEES is transitioning the PP exam from a pencil-and- paper format to a computer-based format consisting of multiple-choice questions is to help eliminate subjectivity in grading, so that the exam papers are consistently graded across groups of examinees. Additionally, a computer-based format consisting of objective multiple-choice questions allows the difficulty of the exam to be psychometrically evaluated for purposes of determining the passing score for a particular administration of the exam. To that point, because computer- based multiple-choice exams are objectively scored, exams offered at different times during the year are able to be compared, or equated, for purposes of setting the passing grade for a particular exam administration.24 Respondent entered into a contract with NCEES in 2009, pursuant to which NCEES provides the FE and PP exams for engineer licensure in Florida. In 2013, the contract was amended to allow NCEES to provide the exams by computer-based testing, using Pearson Vue as its exam 24 As Miller explained,"[i]f they were different on a difficulty level, the harder exam, the standard would actually be lowered so that it would be fair across administration so everybody was treated consistently. Or if my exam was less difficult, the standard would be raised. I would have to get more questions right." administering entity. The FE and PP exams are administered by Pearson Vue at its testing centers. NCEES develops model laws and rules that represent best practices with respect to state licensing of engineers. The aim of these model laws and rules is to achieve uniformity and consistency throughout the states and the U.S. territories and protectorates in the licensure of professional engineers. A significant benefit of such consistency and uniformity is the resulting "mobility" for licensed professional engineers—that is, the ability to more easily become licensed to practice engineering in multiple states. The NCEES model laws and rules establish the "model law engineer," which defines and constitutes the standard for minimal competence in a specific engineering discipline for purposes of being licensed as a P.E. in that discipline. The model laws and rules define the "model law engineer" as a person who holds a degree from an engineering educational program accredited by ABET, has four years of active engineering practice experience, and passes the FE and PP exams. The model law engineer standard equates to the competence level of an engineer having four years of engineering experience who is capable of practicing engineering in a manner that protects the public health and safety. This constitutes the minimum competence level that an applicant must demonstrate for purposes of being licensed as a P.E.25 in the 50 states and the U.S. territories and protectorates. Thus, the NCEES PP exam is constructed to test engineering ability keyed to the model law engineer standard. That is, the PP exam is designed to determine the ability level of an applicant for P.E. licensure for purposes of 25 Refer to note 12, supra. The term "minimal competence," as used in the Challenged Rule, is specifically keyed to the "model law engineer" standard for purposes of being licensed as a P.E. It is not meant to indicate or imply that an engineer who does not take or pass the PP exam is per se incompetent, such that he or she is not competent to engage in work constituting engineering, as defined in section 471.005(7). comparing that ability level to that of an engineer having four years' experience who is able to practice engineering in a manner that protects the public health and safety. Examinees having four years of engineering experience after graduation have the highest pass rates on the PP exams. Pass rates for examinees with more or less than four years of experience are lower, typically in proportion to the length of time before or after the four-year experience mark when they take the PP exam. Miller explained, credibly and persuasively, that the reason for the drop-off in PP exam performance after the four-year mark is that "life happens." Engineers gain more experience, and many become specialized in a relatively narrow niche, or move into managerial, non-technical positions. Additionally, because the PP exam does, in part, test subjects that one learns in college, the longer an examinee is out of college, the less subject matter recall in certain areas he or she may have. "Psychometrics" is the specialized field of study concerned with the theory and technique of psychological measurement. Specifically, psychometrics entails the objective measurement of skills and knowledge, abilities, and educational achievement. Among other specialized areas of practice, psychometricians focus on the construction and validation of assessment instruments, and theories, such as item response theory, that relate to psychological measurement. Psychometricians typically have graduate training and all possess specialized qualifications that enable them to engage in objective psychological measurement. PP exams are designed to determine minimal competence in a specific engineering discipline. "Minimal competence" is the minimal amount of knowledge required to practice in that particular engineering discipline in order to protect the public health and safety. For any specific engineering discipline for which it has been determined that a PP exam should be given,26 there is an approximately three-year due diligence period in which subject matter experts in that discipline work to determine the topics that should be tested on the exam. The PP exam for each specific discipline is developed by subject matter experts, who volunteer and meet on a monthly basis to develop, review, and evaluate the questions for the PP exam for that specific discipline.27 The process of determining which topics should be tested on a PP exam, termed the Professional Activities and Knowledge Study ("PAKS") process, is a standard practice used to determine the specific topics to be tested on a PP exam. As part of the PAKS process, a consulting psychometrician28 employed by Pearson Vue; 15 to 20 engineers who are licensed in another engineering discipline; and subject matter experts who may teach a particular engineering discipline for which the PP exam is being developed, work together to develop consensus regarding the specific topics that engineers having four years of experience practicing in that discipline need to know in order to safely practice engineering in a manner that protects the public health and safety.29 26 For a PP exam to be developed for a new engineering discipline, at least ten NCEES- member state engineering boards must request that such exam be developed, and at least one ABET-accredited program in that specific discipline must exist. 27 Over the years of development and administration of the PP exam, hundreds of licensed engineers have provided input regarding the topics that should be, and are, tested in each discipline and the relative weight given to each topic on the PP exam. 28 Pearson Vue's psychometricians who develop, score, and evaluate NCEES's exams have Ph.D. degrees in psychometrics or statistical analysis. 29 Subject matter experts selected to develop the PP exam questions are chosen based on consideration of the type of practice, such as governmental and private practice; gender; ethnicity; length of time of licensure as a P.E.; and geographic considerations. All subject matter experts must be licensed as a P.E. by a state engineering licensure board in order to The consulting psychometrician builds a questionnaire that lists the specific topics identified by the PAKS committee, and distributes an online survey to engineers who practice in the discipline for which the PP exam is being developed. The survey seeks input regarding the relative importance of each specific topic for purposes of testing to demonstrate minimal competence in the discipline. Based on the survey responses from engineers practicing in the discipline, exam specifications are developed. The exam specifications identify each specific topic to be tested on the PP exam, and the number or percentage of exam questions that will address each specific topic within that discipline. The exam specifications must be approved by an oversight committee. Once the exam specifications have been approved, the subject matter experts for that specific engineering discipline for which the PP exam is being developed prepare the PP exam questions—also termed "items"—and review and evaluate them for clarity, demographic neutrality, and other parameters, so that the items will reliably and validly test engineering ability. In computer-based multiple-choice PP exams, the questions are prepared such that for each question, there is only one correct answer and three other plausible, but incorrect, alternative choices. The individual exam questions are reviewed numerous times by the subject matter experts before they are moved into an exam question bank for use on the PP exam. Once the exam questions have been developed and banked for use on a PP exam, a standard-setting committee, consisting of ten to 15 licensed engineers having diverse backgrounds, experience, and demographic features, reviews the exam to determine the minimum passing score—or "cut score"—on the exam. The cut score equates to the ability level of an engineer serve in this capacity. As noted above, over the years of development and administration of the PP exam, hundreds of engineers have provided input in developing each PP exam. having four years of experience who is minimally competent to practice engineering at a level that protects the public health and safety. This method of using subject matter experts to examine the content of each exam question and predict how many minimally-qualified examinees would answer each question correctly is termed the "Modified Angoff Method." The standard-setting committee then takes the exam, and the psychometrician analyzes the data from the committee's exam sitting. Using this data and analysis, the standard-setting committee then reviews, and reaches consensus, regarding each question, for purposes of determining the proportion of minimally competent engineers who would answer that specific question correctly. Based on the information generated by this process, the psychometrician develops the "panel recommended passing score," with a statistical margin of error. The psychometrician presents this recommended passing score to a committee of five persons, consisting of two state licensing board members and three subject matter experts who observed the exam development process. Based on the psychometrician's recommendation, the committee makes the final decision regarding the minimum passing score for the exam. Each PP exam question is developed and evaluated using the process described above, and is placed in bank for use on a PP exam. The psychometrician uses item response theory to "calibrate"—i.e., determine the relative difficulty level of—each exam question.30 An exam question is not banked for use on future sittings of the exam unless it has had at least 200 responses on a previous exam, so that statistics for each item's performance can be generated for purposes of item calibration. 30 Item response theory is one of many psychometric methods, or tools, used to weight exam questions for purposes of creating different forms of exams having the same level of difficulty. Depending on the specific discipline, a question bank for a PP exam may consist of many thousands of questions.31 Using item response theory, the psychometrician converts the passing score to create a scale from -5 to +5, which will equate to the examinee's ability level as measured by the exam. Once the passing score for the PP exam has been determined, different PP exam "forms" are created for administration in different exam sittings. Exam "forms" are essentially different versions of the PP exam that consist of different individual questions of the same difficulty level, as determined using item response theory, for each specific topic on the exam. Thus, if a PP exam was administered, for example, in April and October, the different exam administrations would consist of different forms—meaning that the exam would consist of different questions, but the questions would be of the same difficulty level for a specific topic tested on the exam.32 Additionally, because exam item difficulty has been calibrated using item response theory, different forms of a PP exam can be given during the same exam administration at different locations.33 Importantly, because the difficulty of the exam items has been calibrated using item response theory, the different exam forms are statistically equivalent in difficulty. 31 The only items that will be used on the graded portion of the PP exam are questions that have known statistics such that they have been calibrated for difficulty. However, there may be other "pretest" questions on the exam strictly for purposes of gathering statistics regarding performance on the questions for potential inclusion in the exam item bank; these "pretest" exam questions are not graded for purposes of determining the examinee's score on the exam. 32 As Miller explained, for an administration of an exam at different locations at the same time, the form administered at a particular location consists of different questions than the form administered at another location; however, the exam forms are equivalent in terms of the number of questions addressing a particular topic and the difficulty of the items addressing that topic. 33 Using the "linear-on-the-fly" ("LOFT") method to generate different forms of the exam also helps ensures exam security, since persons sitting near each other during an administration of the exam will not have the same exam form. As a result of using these processes, including the Modified Angoff Method, and applying item response theory to calibrate the exam items for purposes of constructing different, but statistically equivalent, forms of the PP exam, examinees are not graded on how they perform against each other, but instead are graded against the cut score set for the exam. To ensure that different forms of the exam are statistically equivalent in difficulty, Pearson Vue uses the LOFT method,34 which employs an algorithm to ensure that, across all of the exam forms, all examinees get the same number of questions of the same level of difficulty on the same topics. The algorithm randomly generates, or assembles from banked exam questions, different exam forms based on the exam specifications (i.e., the specific topics tested and relative weight/number of exam questions for that topic) and the difficulty level of the exam questions, such that the different exam forms generated by LOFT are statistically equivalent to each other. Using item response theory to calibrate specific exam question difficulty based on the statistical probability of being answered correctly enables examinees taking different, but statistically equivalent, forms of the exam to be compared to the passing standard for purposes of determining whether they pass the exam. Thus, examinees are compared to an ability level—here, minimal competence—rather than to each other.35 This method ensures that all examinees take an exam of equivalent difficulty, which, in turn, helps ensure the fairness of the exam. 34 For the engineering disciplines having too small a group of examinees to employ item response theory or LOFT to generate different exam forms, each examinee takes the same exam instead of taking different forms of the exam, and the exam typically is offered only on one day, rather than multiple days, per year. 35 By way of example, Miller explained that if two examinees each answer five questions on the same topic on the exam, and one examinee answers four easier questions correctly and the other examinee answers two comparatively more difficult questions correctly, the examinee answering the two more difficult questions correctly may have a higher ability level on that particular topic, due to the comparative difficulty of the questions that examinee answered correctly. Once a PP exam is administered, Pearson Vue scores the examination and sends NCEES the information regarding whether each examinee has passed or failed the exam. Pearson Vue also provides each examinee the information regarding his or her performance on the exam compared to the minimum competence standard. The examinee's performance is expressed as a scaled score, for each specific topic tested on the exam, and for the entire exam. Specifically, using psychometric statistical methods, the ability level of the examinee is expressed as "theta," and is placed on a scale of 0 to 15 for each of the specific topics tested. The examinee's overall theta across all specific topics tested is then compared to the "minimal competence" passing standard, which is also expressed as a scaled score using the same 0 to 15 scale. After an exam is taken and scored, the consulting psychometrician analyzes this data, called "response data," for each exam question, for each examinee, to calibrate the items for purposes of determining whether a particular question performs well in discriminating ability level of the examinees. The psychometrician may recommend that an exam question be "retired" because it is not performing as a good discriminator of ability level. Examples are where an exam question is too difficult or too easy, such that it does not discriminate well in determining ability level; where an item takes too long to answer or is ambiguous; where an item has been "overexposed" by having become publicized such that future examinees have access to the question and scores on the question become high; or where an exam contains "bad pair" items, such that the answer to one item may suggest, or lead to, the answer on another similar item. Having a psychometrician involved in tracking and analyzing exam data enables such circumstances and situations—which may influence the scores on a test item for reasons not related to the examinee's ability—to be identified and corrected. Returning to a free response, paper-and-pencil exam format for the P.E. licensing exam would provide a far less objective, fair, and accurate measure for determining minimal competence for purposes of being licensed to practice engineering. It also would negatively affect the ability of licensed engineers to become licensed in other states. Due to the use of psychometric methods in developing and scoring, the PP exams are very reliable across multiple administrations of the exam— to the point that all of NCEES's psychometric-based PP exams score upwards of .9 on a scale of 1.0.36 Psychometric methods, including item response theory, are used in developing, administering, and scoring many different types of high-stakes professional and academic examinations, including medical school admissions examinations, and nursing, medical examiner, internal auditor, and architecture licensure examinations. NCEES audits approximately one-third of the exams administered by Pearson Vue on an annual basis, to independently evaluate the accuracy of the psychometric services provided by Pearson Vue, and to ensure that the exams have been created pursuant to NCEES's guidelines, procedures, and requirements. NCEES also retains independent psychometricians to review Pearson Vue's exam-related reports and analyses, to ensure that Pearson Vue is following standard psychometric rules of good practice. In sum, the use of objective psychometric methods, including the methods discussed above, to develop, score, and evaluate the PP exam ensures that minimal competence, for purposes of licensure as a P.E., is accurately measured by the exam. Role of the PP Exam in Licensure of PEs in Florida As discussed above, to be licensed as a P.E. in Florida, an applicant must have a college degree from an ABET-accredited institution, four years of 36 Test reliability refers to the degree of consistency with which a test measures a particular subject matter across different administrations of the test. A test has a high reliability score if it consistently produces similar results under consistent conditions. A 1.0 reliability score reflects perfect consistency in results across different administrations of a test. An acceptable reliability score target for high-stakes tests is .7 or higher. active experience in engineering practice, and have passed the FE and PP exams. Thus, the P.E. exam is a vital component of determining that an engineer licensed as a P.E. to practice in Florida is able to practice at a competence level that protects the public health and safety. Unlike the education and experience requirements for licensure— both of which may entail a great deal of variability in quality and breadth across applicants—the PP exam constitutes an objective, consistent tool37 to measure an applicant's level of competence for purposes of determining whether the applicant possesses the minimal competence needed to practice engineering in a manner that protects the public health and safety. As such, the PP exam constitutes a uniform measure of minimal competency for purposes of licensure as a P.E. in Florida. As discussed above, this does not mean that a person who engages in engineering work but has not passed the PP exam is incompetent; it merely means that he or she has not demonstrated minimal competency on this required objective measure of competency for licensure purposes. As discussed above, the PP exam is specifically designed to ensure that a licensed P.E. is competent to practice over a range of specific areas encompassed within a particular discipline. This is because a licensed P.E. is authorized to practice engineering within any discipline or area, subject to professional and ethical standards. The breadth of the PP exam thus helps ensure minimal competence to practice engineering in a manner that protects the public health and safety. 37 This consistency and uniformity is the direct result of the psychometrically-based exam development, scoring, calibration, and validation methods discussed above. The purpose of P.E. licensure is to inform and protect the public, which is entitled to rely on such licensure as indicating that the licensee is competent to practice engineering.38 Administration of the PP Exam During the Covid-19 Pandemic As discussed above, Petitioner has alleged that the Challenged Rule is arbitrary on the basis that it does not address contingencies for offering the exam if unforeseen circumstances prevent regular administration of the PP exam. Specifically, Petitioner points to the fact that Pearson Vue cancelled the April 2020 PP exam administration due to the Covid-19 pandemic. Pearson Vue has rescheduled the PP exams for various times and at various locations around the country in an effort to make the PP exam available for prospective examinees during the pandemic.39 Pearson Vue also is taking substantial steps to protect persons who have applied to take the PP exams during the Covid-19 pandemic. To that point, Pearson Vue has retrofitted its testing centers to help ensure the safety of the examinees as they sit for the PP exam. Specifically, the number of examinees who will be in a testing room has been reduced; masks are required to be worn by examinees and proctors; testing stations are cleaned between each use; some additional test center locations have been added; and some states have relaxed rules to allow examinees to sit for the exam in states other than the one for which they are applying for licensure. At present, the exams are not able to be offered over the internet so that examinees are able to take the exam at a remote location. A substantial reason for this is lack of exam security, which is necessary to protect and 38 As noted above, a person does not have to be licensed as a P.E. to engage in engineering work in Florida. However, if a person wishes to hold himself or herself out to the public as a P.E., then that person must satisfy the requirement to pass the PP exam, which is an indicator of minimal competence for purposes of licensure. 39 Among other things, Petitioner alleges, in paragraph 19 of the Rule Challenge Petition, that the Challenged Rule is invalid under section 120.52(8)(d) because it does not address circumstances where an examination cannot be administered due to force majeure. As maintain the exam's integrity. Additionally, the lack of guarantee of internet service reliability and functionality for every examinee is a crucial consideration, since failed internet connections could significantly affect the fairness of the exam.

Florida Laws (15) 120.52120.54120.541120.542120.56120.569120.57120.68455.217471.005471.007471.013471.01590.70190.702 Florida Administrative Code (8) 1-1.01061G15-18.01161G15-19.00161G15-20.001061G15-20.00261G15-21.00161G15-21.00461G15-30.002 DOAH Case (1) 20-3075RX
# 3
ALBERT POZA vs. BOARD OF ARCHITECTURE, 81-002764 (1981)
Division of Administrative Hearings, Florida Number: 81-002764 Latest Update: May 18, 1982

Findings Of Fact The Petitioner, Albert Poza, applied for licensure by examination to practice architecture in the state of Florida. The architectural licensure examination administered by the Respondent consists of two portions, the written examination given in December of each year and the site planning and design portion administered in June of each year. The Petitioner has complied with all requirements for admittance to the subject examination. The Petitioner sat for a twelve hour examination consisting of a drafting or sketching problem concerning which he was required to design a particular type of building to be accommodated to a particular site, taking into consideration numerous design and site considerations such as human traffic flow, parking, access to all floors, heating and cooling, including natural heating and lighting and numerous other aesthetic, engineering and legal requirements. The examination is administered by the office of Examination Services of the Department of Professional Regulation and is supplied to the state of Florida as well as to all other jurisdictions in the United States by the National Counsel of Architectural Registration Boards (NCARB) . Pursuant to the authority delineated below, this examination has been adopted for use by Florida applicants for licensure. The examination itself is so constituted as to require the applicant for licensure, the Petitioner, to design a structure for placement on a particular site, including mandatory requirements for accommodating the structure to the site, and vice versa, detailed design of elevations, building cross-sections, facades, and floor plans, as well as effective use of natural light and solar heating potential, regard for the physical and aesthetic needs of the building's occupants, its impact on the environment of the site and its locality and numerous other criteria. Prior to sitting for the examination, each applicant, including the Petitioner, receives a pre-examination booklet setting forth the architectural program to be accomplished by the applicant and various requirements to which the Petitioner is expected to apply himself in order to receive a passing grade. Immediately prior to commencing the examination itself, the Petitioner received other information designed to enable him to more adequately design the structure requested and perform the necessary technical and architectural requirements of the problem. In general, the examination was designed to require the Petitioner to design a solution to the site plan and the building design problems submitted to him by NCARB. The pertinent portion of the examination thus allows the examination graders, and through them, the Florida Board of Architecture, to determine whether an applicant such as the Petitioner is able to coordinate the various structural design, technical, aesthetic, energy and legal requirements in order to resolve the design and site plan problem after having been tested on these same requirements in written form in the initial portion of the examination administered in December of each year. The grading of the site and design portion of the examination was accomplished by submission of the Petitioner's work product to at least three architects selected by the various architectural registration boards of some twenty states. These graders are given training by the NCARB in order to standardize their conceptions of minimal competence required for achievement of a satisfactory grade on the examination. Each architect grader is then asked to review and score various solutions to the site and design problem submitted by applicants, including the Petitioner, on a blind grading basis. The grader has no knowledge of the name or state of origin of the applicant whose solution he is grading. The grader is instructed to take into consideration the various criteria set forth in Rule 21B-14.03, Florida Administrative Code, and the evaluation criteria set forth in the grading sheet. The graders are instructed to note areas of strength and of weakness in an applicant's solution with regard to the grading criteria and then determine, based on an overall conception of the solution submitted by the applicant, whether or not a passing grade is warranted. A passing grade is defined as a holistic grade of three or four as set forth in Rule 21B-14.04, Florida Administrative Code. The applicant must receive at least two passing grades from the three architect graders who independently grade his solution to the problem in order to pass the relevant portion of the exam. The Petitioner herein received two "2's", which are failing grades and one "3", which is a passing grade. The Petitioner demonstrated an effort to comply with the instructions set forth in the examination, as well as the pre- examination booklet. He failed, however, to achieve sufficient clarity of presentation in several material areas such that the graders could make a clear determination that he understood and had complied with sufficient of the mandatory criteria to achieve passage of the examination. As established by Herbert Coons, Executive Director of the Florida Board of Architecture, and Glenn Paulsen, Professor of Architecture at the University of Michigan, both graders of the Petitioner's examination and the Respondent's expert witnesses, the Petitioner failed to supply sufficient information to permit a passing score to be awarded based upon the criteria required to be considered and complied with in the examination program and by the authority cited below. The Petitioner's examination was deficient in a number of material respects. The Petitioner violated the setback requirements as to the side entrance of the building in question with the result that significant alterations of the off-site and publicly owned sidewalks and easements would be required in order to effect his design solution. It is not good architectural practice, when asked to design a structure, to use land which is not part of the land owned by the client requesting the design. The examination program also specifically required that the candidate either maximize the floor space in the building by eliminating some amenities or if determining to insert amenities such as atria, balconies, large interior spaces and so forth, that these be provided in such a way to make the structure a luxury-type office building. In effect, the owner's goals in this design problem sought either a functional building with maximum floor space or a luxury building with minimal floor space, but with significant cultural amenities. In his solution, the Petitioner did not meet either of those two goals, since he included minimal floor space and yet an insufficient number of luxury features required by the program as an alternative. Other significantly deficient areas in the Petitioner's examination solution included his failure to visually relate the building's design to adjacent buildings. That is, he ignored the instructions in the program requiring him to design a building in an area of historical significance with an appearance which is compatible with adjacent historical buildings; pictures of adjacent buildings being furnished in his examination booklet. The Petitioner failed to allow adequate room in the mechanical equipment space for heating and air conditioning equipment, which the size and type building would require. Additionally Witness Coons established, by scaling the Petitioner's design solution, that the building was too large for the site on which it was to be constructed. Portions of it would encroach upon public property and violate local zoning ordinances. In a more serious vein, it was established that the building design did not contain an adequate allowance for structural support as to the various spans over the columns. The column spacing was appropriate, but the beams depicted are not of a sufficient size and type to safely support the structure and there is a danger that a building so constructed would collapse. Additional deficiencies noted involved poor human traffic circulation in the third floor lobby area design, insufficient storage space included in the design for the third floor lecture room and inadequate provision for landscaping. Other less significant deficiencies were noted including, as admitted by the Petitioner that the square footage on the upper floors was in error. In short, significant program requirements were not provided for or complied with by the Petitioner. In view of the above determined deficiencies, the Petitioner failed to establish that his solution to the site and design problem posed by the examination reflects sufficient and appropriate consideration of the requirements and criteria he was instructed to address.

Recommendation Having considered the foregoing findings of fact and conclusions of law, the evidence in the record, the candor and demeanor of the witnesses, and the pleadings and arguments of the parties, it is, therefore

Florida Laws (5) 120.57455.217481.209481.211481.213
# 4
WILLIAM T. DAVENPORT vs CONSTRUCTION INDUSTRY LICENSING BOARD, 94-003534 (1994)
Division of Administrative Hearings, Florida Filed:Jacksonville, Florida Jun. 28, 1994 Number: 94-003534 Latest Update: Apr. 11, 1995

Findings Of Fact Test Anxiety at the February 1994 General Contractor's Examination The General Contractor's Examination used in the licensing of individuals as general contractors is divided into three parts, each of which may be passed separately. Credit for passing any one portion of the examination is good for a period of only one year. An applicant for a license as a General Contractor is limited in taking any part of the examination to three times per year. If an applicant fails to pass all three parts of the examination in one year, therefore, the applicant must commence the examination process anew, as if none of the portions of the examination had been passed. On February 17, 1994, William T. Davenport, sat for the Contract Administration portion of the exam for the third time within a one-year period. At each of the two examinations taken earlier, he had passed one portion. The Contract Administration portion was the only part he had not yet passed. Mr. Davenport was anxious as he waited to take the exam. He fully realized that if he did not pass the Contract Administration portion he would have to start the examination process all over again. His concern was generated not only from the perspective of delay and inconvenience attendant to having to repeat the entire exam process, but also from the perspective of cost. Repeating the process would require him to pay the full exam fee after having paid already a full examination fee for each of three examinations in the year of his attempt to successfully complete the examination requirements for licensure. Different Approved Reference Lists The Construction Industry Licensing Board approves reference materials that applicants may use during the examination and issues a list of those materials periodically. The Reference List for the February examination covered the period between November 1, 1993 and February 28, 1994. On that list was a book Mr. Davenport brought with him to the examination: Practical Mathematics, 3rd Edition, Copyright 1972. As Mr. Davenport sat nervously awaiting commencement of the exam, a proctor removed Practical Mathematics from his desk indicating that use of the book was not allowed. Mr. Davenport relates his response to the incident in this way, (Tr. 26.) The anxiety was very high at that point. My concern was try to get through the exam and, when the book was taken away from me, I panicked, to be blunt. I just totally panicked. Unlike the testing period from November 1, 1993 to February 28, 1994, Practical Mathematics was not on the Reference List for the next period of testing, from March 1, through June 30, 1994. The later list, issued January 13, had been out for over a month at the time of the exam. It is likely the proctor removed the book in mistaken reliance on the later list. As it turned out, the proctor returned the book to Mr. Davenport either shortly before the examination commenced. As he did so, the proctor commented, "Well, I don't know." (Tr. 25.) Mr. Davenport did not use the book during the exam because, "[a]t that point, I was reluctant to use the book ... I didn't want the test to be invalidated and I didn't want to be challenged." Id. Not using the book proved to be critical to whether Mr. Davenport passed the Contract Administration portion of the exam. He scored a 68. A passing grade is 70. Had Mr. Davenport answered correctly question number 3, which was worth four points, he would have received a 72, a passing grade. The question involves applying a percentage. Mr. Davenport could not remember whether in obtaining a percentage it is necessary to multiply or divide. Practical Mathematics has a chapter on percentages. The chapter teaches that multiplication is the arithmetic method to use when obtaining a percentage. But Mr. Davenport guessed that division should be used. He divided by the percentage and, therefore, chose an incorrect answer. Question number 3 on the exam is one of the questions that Mr. Davenport challenged originally: A 2-man crew has consistently worked at a labor performance standard ratio of 0.85 to 1. They are selected for a job requiring 60 (standard time) man-hours to perform. They will NOT work more than 8 hours per workday. NO work will be done on Saturdays or Sundays. There are NO holidays during the time the work will be performed. According to Builder's Guide to Accounting, if the job must be finished NO later than Friday afternoon at 5:00 p.m., what is the last day that they could be scheduled to start the job? Thursday of the previous week Friday of the previous week Monday of the same week Tuesday of the same week Respondent's. Ex. No. 2. The correct answer is "(D) Tuesday of the same week." The Department's expert witness explained that the correct answer is reached by way of an algebraic formula. The formula is: "the labor performance standard ratio = x (the unknown) divided by the standard time man hours" and then that answer is divided by 2 since the crew is a 2 man crew. Applied to the problem, the formula is: .85/1 = x/60, with x, once known, divided by 2. Using the formula, the calculation goes as follows: .85/1 = x/60; multiplying both sides of the equation by 60, .85(60) = x; carrying out the arithmetic calculation, x equals 51; 51/2 = 25.5. It takes 25.5 hours, therefore, for the crew to complete the job. If, as the problem states, the job must be completed by Friday at 5 p.m. and the crew works 8 hours a day, then it will take the crew 3 full days and 1.5 additional hours to complete the job. Working backward from Friday, the crew will work 8 hours on Friday, 8 hours on Thursday, and 8 hours on Wednesday for a total of 24 hours. The crew must start on Tuesday of the same week to work the additional 1.5 hours required to complete the job. One does not need to use algebra, however, to solve the problem. One can simply obtain the number of actual hours needed to complete the job by applying 85 percent (the crew's labor performance standard) to the number of standard time manhours called for by the job, in this case, 60. Here is where Mr. Davenport needed Practical Mathematics. Not knowing whether to obtain the actual hours by multiplying .85 times 60 or dividing .85 into 60, he guessed, in error, division. Through the use of division, it appears incorrectly that the number of manhours needed is 70.588. Divided by two, to take into account that there are two members of the crew, it would take the crew 35.294 hours. If it took the crew 35 hours and a fraction to complete the job on time, the crew would need to start on Monday of the same week. "(C) Monday of the same week," is the answer chosen by Mr. Davenport. Other Distractions and the Site of the Exam The examination was conducted in the Tallahassee office of the National Assessment Institute (NAI). The NAI was under contract to the Department as the vendor to conduct the exam. At the time of the examination, the exam site was a room approximately forty feet by thirteen feet four inches in size. It contained nine tables, each 18 inches by eight feet. The tables were spaced 35 inches from each other. At each table were two straight-backed chairs. There were seven candidates present for the examination. The first and last tables were unoccupied. Each of the seven candidates were allotted 34 square feet of floor space to be occupied by the candidate, the chair, the table and materials used in the exam. On the east wall of the exam room were three plate glass fixed windows. Two of the windows are 48 inches long by 36 inches tall and one is 36 inches long by 31 inches tall. The glass is one-eighth inch thick non- commercial grade. The windows are acceptable under NAI guidelines. On the other side of the windows is a workroom that measured 15.25 by 17.5 feet. Through these windows the examination supervisor seated in the work room can monitor the performance of the proctors in the exam room and see the candidates as they take the exam. The candidates, seated to the left of the workroom, do not face the workroom. Rather, their right side is exposed to the workroom. If the tables are numbered 1 through 9 on Respondent's Ex. No. 1, beginning with 1 on the side of the room marked on the exhibit as "N" or north, candidates who were seated at tables 4 through 8 were directly exposed to the workroom windows. Mr. Davenport was seated at one of the tables exposed to the workroom windows, most likely table 3, 4 or 5, that is, one of the center 3 tables. During the exam, he could see employees through the windows moving in the workroom and hear noise from the workroom. There were four employees who were present at one time or another in the workroom. Three of these employees were also engaged in proctoring the examination. In addition to the visual diversion posed by the four employees in the workroom, Mr. Davenport could hear sounds emanating from the room. The doors to the workroom, open so that the exam supervisor seated in the workroom could hear what occurred in the exam room, also allowed sounds from the workroom into the exam room. The source of the sound was the printer working, the four workers conversing from time to time and other noises associated with an office work environment. Sight of the employees and noise from the workroom prevented petitioner from fully concentrating on the exam. Mr. Davenport was also distracted by the activities of the proctors while in the exam room. During the four hours he sat for the exam, three of the four employees he observed in the workroom were also acting as proctors. They left the workroom in a rotation in order to spell each other. During their shifts as proctors, the three monitored the exam room. Mr. Davenport felt distracted by the coming and goings of the three as they rotated in and out of the room. Although there was a table designated for the proctors at a corner outside the workroom across from table 3, they rarely sat there. They sat at one of the empty tables or walked beside the seated candidates, all the time carrying out the function of a proctor: observing the candidates during examination. The FCILB Examination Administration Manual, applicable to the February General Contractor's exam, details the responsibilities of proctors in sixteen separate counts. No. 13 reads: Proctors observe at all times and move quietly about the room. Proctors do not disturb or distract candidates during the examination. If speaking is necessary, a proctor needs to be quiet and brief as possible. Proctors avoid asking candidates to move chairs to get around them, standing too close or directly behind candidates, or rustling papers and talking to other proctors in the vicinity. Petitioner's Ex. No. 2, FCILB Examination Administration Manual, p. 2-5. Movement of the proctors was necessary during the exam because of its open-book format. It is incumbent on the proctors of an open-book exam to insure that candidates do not copy questions form the examination into their reference materials. Other Candidates Reactions to the NAI's Tallahassee Office Among the three proctors the day of the exam was Ms. Jean Love. Ms. Love is also the Office Manager of the NAI's Tallahassee Office. She has worked for NAI for over two years. Before that she worked for eight and one-half years with the Department in examination services, during which she administered exams, including acting as a proctor for exams. In addition to the daily operations of the office, she oversees the administration of examinations, a function she fulfilled at the February General Contractor's exam this year. Ms. Love did not see any unusual or distracting activities on the part of the other two proctors and did not undertake any activities, in her opinion, that would have violated any of the responsibilities of proctors, including those quoted, above, from the FCILB Examination Administration Manual. The activities during the exam in the workroom, undertaken under Ms. Love's supervision, were normal activities undertaken every day at the NAI Tallahassee Office during and outside of times of examinations. Aside from typical office activities, such as conducting telephone conversations, scheduling candidates for tests, and doing paperwork that included hand-folding documentation, there was no unusual activity the day of the exam. The only event in the workroom that contributed at all to the sound of normally quiet office activity was the validation of a single candidate's check. No complaints about noise in the workroom during the February General Contractor's Exam were registered with the NAI Tallahassee Office. Nor did any of the candidates that day complain about the activities of the proctors. Ms. Love did not learn of Mr. Davenport's complaint until after he filed his challenge to the examination questions. While a proctor may have from time to time stood near Mr. Davenport as he took the exam, none of the proctors hovered over him or, in Ms. Love's opinion, did anything that would distract the average candidate. No complaint during the examination was made by Mr. Davenport. He did not complain about inability to concentrate on the exam until after he received the exam results. During Ms. Love's two years at the NAI Tallahassee office, no candidate, prior to Mr. Davenport, had ever complained about the testing environment for any reason. The comments she has received from candidates following exams have been solely complimentary. Over the last two years, the office has administered between 15 and 20 tests per month. Complimentary comments are made, on average, by one candidate per test. In the last two years the office has received, at a minimum, well in excess of 350 compliments on the testing environment from candidates. In contrast, Mr. Davenport's complaint stands alone as the only complaint about the office testing environment in the last two years at the NAI's Tallahassee Office.

Recommendation It is, accordingly, RECOMMENDED, in the alternative: That petitioner's request for reexamination or a passing grade on the "Contract Administration, Division I" portion of the General Contractor's examination administered in February 1994 be DENIED; or, in the alternative, If the Construction Industry Licensing Board is willing to overlook the petitioner's failure to challenge his grade specifically on the appropriation of the book before the examination in the petition for formal hearing and the Department's legitimate objection to the presentation of evidence on the issue, that petitioner be allowed to sit for reexamination and, if he passes the Contract Administration portion of the exam, be credited with passing the other two portions of the exam as well as if all three portions had been passed in one year. DONE and ENTERED this 1st day of December, 1994. DAVID M. MALONEY Hearing Officer Division of Administrative Hearings The DeSoto Building 1230 Apalachee Parkway Tallahassee, Florida 32399-1550 (904) 488-9675 Filed with the Clerk of the Division of Administrative Hearings this 1st day of December, 1994. APPENDIX Petitioner's proposed findings of fact Nos. 3, 4, 5, 6, 9, 10, 12, and 13 are adopted, in substance, insofar as material. With respect to petitioner's proposed finding of fact No. 1, the first and last sentences are adopted. The remainder of the proposed finding, and in particular the reference to Linda Chaffin, test proctor, is rejected because it is either not supported by the evidence or argumentative in nature rather than factual. Petitioner's proposed finding of fact No. 2 is rejected as against the weight of the evidence. Petitioner's proposed finding of fact Nos. 7 and 8 are rejected as unsupported by the evidence. With respect to petitioner's proposed finding of fact No. 11, the reference to Linda Chaffin is rejected. Ms. Chaffin was not identified by the evidence as the proctor who removed the book from petitioner prior to the exam. Respondent's proposed findings of fact are adopted, in substance, insofar as material. COPIES FURNISHED: William T. Davenport 336 14th Avenue, North Jacksonville, FL 32250 William W. Woodyard Assistant General Counsel D B P R 1940 North Monroe Street Tallahassee, FL 32399-0750 Richard Hickok, Executive Director C I L B 7960 Arlington Expy., Ste. 300 Jacksonville, FL 32311-7467 Jack McRay General Counsel D B P R 1940 North Monroe Street Tallahassee, FL 32399-0750

Florida Laws (2) 120.57489.111
# 5
ROGER S. EVANS vs BOARD OF PROFESSIONAL ENGINEERS, 91-001580 (1991)
Division of Administrative Hearings, Florida Filed:Tampa, Florida Mar. 12, 1991 Number: 91-001580 Latest Update: Aug. 20, 1991

The Issue Whether Petitioner's application for licensure by examination as an engineering intern should be granted.

Findings Of Fact Prior to his admission to the Mechanical Engineering Program at the University of South Florida on August 30, 1982, Petitioner Evans attended a three-year full-time Mechanical Engineering Diploma Program at the College of Arts, Science and Technology in Kingston, Jamaica. Upon completion of the program, Petitioner was awarded the College Mechanical Engineering Diploma. The diploma from the College of Arts, Science and Technology was conferred in an educational system based upon the English System of Education. The diploma was not a university degree, such as a Bachelor of Science. It is more akin to a certificate from a specialized training program. Such diplomas are often called Associate Degrees when they are issued by junior colleges in the United States. 750 credit hours were transferred from the College of Arts, Science and Technology and were applied to the lower level requirements for the Mechanical Engineering Program when Petitioner was enrolled at the University of South Florida. As with all transfers from other schools of higher education, Petitioner was not given credit for those courses in the grade point average (GPA) he was required to achieve at the university. Throughout his enrollment at the university prior to the actual award of his Bachelor of Science (BS) degree, Petitioner Evans was in the Mechanical Engineering Program. During the thirteen terms the Petitioner attended the university before he was awarded his BS degree, he repeated the following engineering department courses: EGN 3313 STATICS (3 times); EML 4503 MACH AN & DES 2 (2 times); ENG 4314 AUTO CONTROLS I (3 times) and EML 4106 C THERM SYS & ECO (4 times). Petitioner ultimately achieved a "A" in EGN 3313 STATICS; a "C" in EML 4503 MACH AN & DES 2, as well as ENG 4314 AUTO CONTROLS I. His final grade in the coursework for EML 4106 C THERM SYS & ECO was a "B". At all times while Petitioner was in attendance at the university, the Mechanical Engineering Department required students to have a GPA of 2.2 or better in a specific schedule of coursework before a Bachelor of Science in Mechanical Engineering (BSME) degree would be awarded by the faculty of the Department. The curriculum for the Mechanical Engineering Program at the University of South Florida was accredited by the Accreditation Board for Engineering and Technology (ABET) based upon the program requirement that a degree in mechanical engineering would be conferred only on students with a 2.2 or better GPA. The fall term of August 24, 1987 - December 12, 1987, was designated as Petitioner's final term of his senior year as an undergraduate seeking a BSME degree. Although the means used by the Mechanical Engineering faculty to calculate a GPA during this particular time period was unavailable, there is no dispute that the faculty applied its policy and determined that a BSME could not be awarded to Petitioner because he did not meet the academic standard of 2.2 or better GPA in the scheduled courses. Due to the averaging required to arrive at a GPA, Petitioner's repetition of so many courses lowered his overall GPA even though he successfully completed each course on his final attempt. When Petitioner was personally informed of the faculty's decision by his assigned faculty adviser, he questioned whether he could retake some of the courses to bring his GPA status up to the level demanded by the faculty. This idea was discouraged by his adviser because Petitioner would have to repeat a large number of courses over a lengthy period of time. The averaging techniques used to compute a GPA makes such an endeavor very time consuming with small results for the effort spent. Based upon the advice he received, Petitioner acquiesced in the faculty's decision to award him a B.S. in Engineering-Option in General and accepted the degree. At the close of his undergraduate academic pursuits, Petitioner had an overall GPA of 2.082 and a GPA in departmental course work of 1.79. This departmental GPA was calculated by eliminating 3 "Fs" from his transcript, per the university's forgiveness policy. All other course repeats lowered his overall GPA and his departmental GPA. In spite of the overall GPA and departmental GPA determination, Petitioner did take and successfully passed every course within the curriculum of the Mechanical Engineering Program at the University of South Florida. The B.S. degree awarded to Petitioner is an alternate degree within the university. It is designed for students who have either completed a specialized program but were unable to meet a faculty's higher GPA standard or for those students who never designated a specialty within the engineering school, but met general university degree requirements. This program has never been accredited by ABET. ABET relied upon the faculty's representation that students who received BSME degrees would obtain a 2.2 or better GPA in the program before the degree was awarded when accreditation was granted by the board. It is unknown as to whether the program would have been approved if a lower success standard had been set for the students. On July 9, 1990, Petitioner's application for the Fundamentals Examination was received by the Department. The application was rejected on September 24, 1990, because the Department determined Petitioner did not meet the statutory and rule provisions governing admissions to the examination. From August 27,, 1984 - December 11, 1987, Petitioner was in the final year of an approved engineering curriculum in a university approved by the Board. He successfully completed the courses in the curriculum, but his GPA in the program was lowered by his numerous repetitions of the same courses before successful completion occurred.

Recommendation Based upon the foregoing, it is RECOMMENDED: Petitioner's application to take the examination administered by the Department for the Board be denied. DONE and ENTERED this 20th day of August, 1991, in Tallahassee, Leon County, Florida. VERONICA E. DONNELLY Hearing Officer Division of Administrative Hearings The DeSoto Building 1230 Apalachee Parkway Tallahassee, Florida 32399-1550 (904)488-9675 Filed with the Clerk of the Division of Administrative Hearings this 20th day of August, 1991. APPENDIX TO RECOMMENDED ORDER Petitioner's proposed findings of fact are addressed as follows: Pages 1-2: Accepted. See Preliminary Statement Issue I-Page 3: Paragraph one. Accepted. See HO #11. Paragraph two. Accepted. See HO #7. Paragraph three. Accepted. See HO #3. Paragraph four. Accepted. See HO #8. Paragraph five. Accepted. See HO #4, #10, #11 and #12. Paragraph six. Accepted. Paragraph seven. Accepted. See HO #15. Paragraph eight. Accepted. See HO #12. Paragraph nine. Accepted. Paragraph ten. Accepted. Paragraph ten. Rejected. Cumulative. Issue II-Page 7: Paragraph one. Accepted. See HO #13. Issue III-Page 8:Paragraph one. Accepted. Paragraph two. Rejected. Cumulative. Paragraph three. Accepted. Paragraph four. Rejected. Mixed Question of Law and Fact. Witness Incompetent to determine. Paragraph five. Rejected. Cumulative. Respondent's proposed findings of fact are addressed as follows: Accepted. See HO #14. Accepted. See HO #14. Accepted. See HO #1. Accepted. See HO #3 and #4. Accepted. See HO #13. Accepted. See HO #12 and #13. Accepted. See HO #8 and #13. Rejected. Contrary to fact. See HO #5. Accepted. See HO #11. Rejected. Irrelevant. Accepted. Accepted. See HO #10. Rejected. Insufficient facts presented. See HO #8. Accepted. See HO #6. Accepted. Rejected. Irrelevant. Rejected. Irrelevant. Rejected. Improper legal conclusion. Rejected. Contrary to fact. See HO #12. COPIES FURNISHED: Weldon Earl Brennan, Esquire SHEAR NEWMAN HAHN & ROSENKRANZ, P.A. 201 E. Kennedy Boulevard, Suite 1000 Post Office Box 2378 Tampa, Florida 33601 Edwin A. Bayo, Esquire Assistant Attorney General Department of Legal Affairs Suite LL04, The Capitol Tallahassee, Florida 32399-1050 Carrie Flynn, Executive Director Jack McRay, General Counsel Florida Board of Professional Department of Professional Engineers Regulation Northwood Centre, Suite 60 Northwood Centre, Suite 60 1940 North Monroe Street 1940 North Monroe Street Tallahassee, Florida 32399-0755 Tallahassee, FL 32399-0792

Florida Laws (5) 120.56120.57455.11471.005471.013
# 6
RONNIE F. TAYLOR vs. BOARD OF PROFESSIONAL ENGINEERS, 87-004137RX (1987)
Division of Administrative Hearings, Florida Number: 87-004137RX Latest Update: Jan. 29, 1988

The Issue The issue is whether Rules 21H-21.002(1) and 21H-21.004(1) are an invalid exercise of delegated legislative authority.

Findings Of Fact Ronnie F. Taylor, of Post Office Box 697, Cedar Key, Florida, is employed by the engineering firm of Ingley, Campbell, Moses and Associates of Gainesville, Florida, which engages in mechanical, electrical and plumbing engineering. Taylor has been with this engineering firm for four years and is currently a vice president in charge of production of electrical engineering documents. Prior to this employment, Taylor spent 14 years as an electrical engineer with the engineering firm of Reynolds, Smith and Hill of Jacksonville, Florida. When Taylor left Reynolds, Smith and Hill, he was the senior design engineer. Taylor served in the military as an electrician. Upon completing military service in 1967, Taylor entered Florida Junior College. He received an Associate of Science degree in Electrical Engineering Technology in 1970 from that institution. Following that degree, Taylor began employment with Reynolds, Smith and Hill, where his responsibilities included the design of electrical projects for commercial buildings, including writing specifications, making cost estimates and producing a finished product. Taylor has spent his entire career in electrical engineering and has no experience with other specialties of engineering. He has extensive experience in electrical engineering having designed and completed numerous large commercial projects. However, because Taylor is not a licensed professional engineer, a licensed professional engineer must oversee all projects during the course of design and completion and must sign and seal all completed work. Taylor is not a licensed professional engineer because he has failed to pass the Fundamentals of Engineering (FE) portion of the engineering examination. He has failed in fourteen attempts to pass the FE exam. Taylor did pass the Principles and Practices (P & P) portion of the exam in 1982. Licensure requirements specify that both sections must be passed prior to licensure. Taylor became qualified to take the engineering exam in 1977 pursuant to Section 471.21(1)(c), Florida Statutes (1977), which permitted an applicant to take the exam with "a specific record of 10 years or more of active practice in engineering work of a character indicating that the applicant is competent to be placed in responsible charge of such work." This so-called 10 year cycle permitted an applicant to qualify for the exam without the otherwise required 4- year college degree and 4 additional years of experience. In 1979, Section 471.013, Florida Statutes, was enacted, allowing persons in the final year of engineering school to take the FE exam to qualify as an engineer intern. This provision has been in effect since 1979. The FE exam, as required by Rule 21H-21.002(1), which is challenged here, includes questions on the subjects of mathematics, mathematical modeling of engineering systems, nucleonics and wave phenomena, chemistry, statistics, dynamics, mechanics of materials, fluid mechanics, thermodynamics/heat transfer, computer programming, electrical circuits, statics, structure of matter, engineering mechanics, electronics and electrical machinery. While Taylor scored highly on the subjects relating to electrical engineering, he had difficulty with other areas of the exam. The course work completed by Taylor in 1970 did not include some of these areas with which Taylor had difficulty. Taylor has had no course work in computer programming, thermodynamics, statistics, nucleonics and wave phenomena. The subjects tested in the FE exam are updated in order to test applicants on the most current information and knowledge of engineering fundamentals. Herbert A. Ingley is a licensed professional engineer and holds a Bachelors degree in Chemical Engineering, a Masters degree in Mechanical Engineering, and a Ph.D. in Mechanical Engineering with a minor in Environmental-Mathematics. He taught full time on the faculty of the University of Florida in Mechanical Engineering for 11 years. In his opinion, it is more difficult for applicants to pass the FE exam the further they are from their formal education and, therefore, applicants in the 10 year cycle have more difficulty passing the exam. According to Ingley, the requirement that persons such as Taylor wait 10 years before taking the FE exam is not logical. However, Ingley also opined that it is important for a professional engineer to have a fundamental knowledge of engineering and that there is a need to test the fundamental basics of engineering for each person who is going to become a licensed professional engineer. George Edward Rabb is a licensed professional engineer, having been licensed in 1965. He was grandfathered and therefore only had to pass the P & P exam. The FE exam was waived based on specific portions of statute and rule which waived the FE exam for persons with fifteen years experience. The waiver was only available to persons qualifying prior to November, 1970. According to Rabb, an engineer needs to have a working knowledge of fundamentals and to understand the general concepts of engineering. Robert D. Kersten, who has been the Dean of the Department of Engineering at the University of Florida for 20 years, has a Bachelors degree in Mathematics and Chemistry, a Masters degree in Civil Engineering, and a Ph.D. in Civil Engineering, Water Resource/Hydrologic Engineering. Dean Kersten has served in numerous capacities with both state and national professional associations involved in accreditation of engineers and served on the Board of Professional Engineers in Florida and on the National Council of Engineering Examiners. The FE exam is prepared by the National Council of Engineering Examiners and is designed to cover the fundamental areas essential to the basic practice of engineering. The FE exam tests both the common body of knowledge that is essential to practice in the profession and the ability to apply that knowledge. According to Dean Kersten the FE exam tests items which should be within an engineer's basic knowledge and which are necessary to communication between engineers in a design team approach to project design. Dean Kersten acknowledges that the FE exam is more difficult for applicants who lack a degree or who have been out of the academic area for a period of time, but opines that those factors do not excuse an applicant from mastering and retaining the basic fundamentals important to the practice. In fact, the FE exam is designed so that 70 percent of the applicants with-the 4- year college educational background pass the exam. Only 40 percent of the applicants in the 10 year cycle pass the exam.

Florida Laws (8) 120.52120.54120.56120.68455.217471.008471.013471.015
# 7
DENNIS VANN vs DEPARTMENT OF BUSINESS AND PROFESSIONAL REGULATION, FLORIDA ENGINEERS MANAGEMENT CORPORATION, 99-004776 (1999)
Division of Administrative Hearings, Florida Filed:Tampa, Florida Nov. 16, 1999 Number: 99-004776 Latest Update: Jul. 17, 2000

The Issue Whether Petitioner is entitled to additional credit for his responses to Question No. 130 of the Principles and Practice of Engineering portion of the engineering licensure examination administered on April 23, 1999, by the National Council of Examiners for Engineers and Surveyors.

Findings Of Fact Petitioner, Dennis Vann (Petitioner), is an applicant for licensure as a professional engineer in the State of Florida. On April 23, 1999, Petitioner sat for the Principles and Practice Engineering Examination portion of the engineer licensure examinations. This is a national examination developed, controlled, and administered by the National Council of Examiners for Engineering and Surveying (NCEES). Petitioner received a raw score of 45 on this examination. For the electrical engineering discipline, a raw score of 45 results in a converted score of 67. A minimum converted score of 70 is required to pass this examination. A raw score of 48 results in a converted score of 70. Therefore, Petitioner needs an additional 3 raw score points to earn a passing score on the examination. Petitioner challenged the scoring of Question No. 130 on the examination and formally requested the NCEES to rescore his solutions to the question. The NCEES rescored Question No. 130 and determined that Petitioner was not entitled to any additional points for Question No. 130. For Question No. 130, the maximum score achievable was Petitioner received a score of 4 on that item. The NCEES developed and used an Item Specific Scoring Plan (ISSP) for each examination question. Question No. 130 was scored by the NCEES according to the ISSP for that question. Question No. 130 contains two subparts, which require the examinee to address four discrete requirements. Petitioner correctly calculated the bus current (requirement 3). However, Petitioner failed to properly calculate the busway loading and determination of adequacy (requirement 1), the bus impedance (requirement 2), and percent voltage drop (requirement 4). Petitioner's response to Question No. 130 was initially assigned a score of 4. However, if graded correctly, that response would have resulted in a score of 6. The credible testimony of Respondent's expert was that under the ISSP for Question No. 130, Petitioner is entitled to a score of 6 for his response. With a score of 6 for Question No. 130, Petitioner's raw score is increased to 47. A raw score of 47 results in a converted score of 69. Even with the 2 additional points awarded to Petitioner's response to Question No. 130, his score on the professional engineering licensure examination is still below 70 and is not a passing score. Question No. 130 provides all the necessary information for an examinee to solve the problem. Moreover, Question No. 130 is properly designed to test an examinee's competence in electrical engineering.

Recommendation Based upon the foregoing Findings of Fact and Conclusions of Law, it is hereby RECOMMENDED that a final order be entered concluding that Petitioner is entitled to a score of 6 points for his response to Question No. 130, and recalculating Petitioner's total score on the examination on the basis of that conclusion. DONE AND ENTERED this 21st day of April, 2000, in Tallahassee, Leon County, Florida. CAROLYN S. HOLIFIELD Administrative Law Judge Division of Administrative Hearings The DeSoto Building 1230 Apalachee Parkway Tallahassee, Florida 32399-3060 (850) 488-9675 SUNCOM 278-9675 Fax Filing (850) 921-6847 www.doah.state.fl.us Filed with the Clerk of the Division of Administrative Hearings this 21st day of April, 2000. COPIES FURNISHED: Dennis Vann Post Office box 23054 Tampa, Florida 33623 William H. Hollimon, Esquire Ausley & McMullen 227 South Calhoun Street Post Office Box 391 Tallahassee, Florida 32301-1884 Dennis Barton, Executive Director Department of Business and Professional Regulation Board of Professional Engineers 1208 Hays Street Tallahassee, Florida 32301 Natalie A. Lowe, Esquire Florida Engineers Management Corporation 1208 Hays Street Tallahassee, Florida Barbara D. Auger, General Counsel Department of Business and Professional Regulation 1940 North Monroe Street Tallahassee, Florida 32399

Florida Laws (2) 120.569120.57
# 8
ARMANDO PEREZ AND MIGUEL OYARZUN vs. CONSTRUCTION INDUSTRY LICENSING BOARD, 75-001231 (1975)
Division of Administrative Hearings, Florida Number: 75-001231 Latest Update: Jan. 17, 1977

Findings Of Fact On May 23, 1975, the Petitioners herein took the general contractors licensing exam in Miami, Florida. Petitioners failed to achieve a passing score on said examination and thereafter several reviews of the examination questions and answers resulted. The exam in question was administered to approximately 659 examinees. The test was made up of 100 questions and the examinees were allotted 4 hours to complete the exam. The examinees were instructed in the examination booklet to answer as many questions as possible within the time limit and to always select the best possible answer out of the listed choices. The examinees were furnished a list of reference materials by Respondent and they were advised that the exam questions would come from some 18 odd reference books supplied on the reference lists. Petitioner Dennis Milch took the general contractor's examination administered on May 23, 1975. 2/ In preparation of the exam he took a prep course given by Cole Construction and began his preparation approximately two months prior to the exam. He received a score of 67.5 on the exam. He earned a degree from the University of South Florida and took advance construction courses at FIU where he earned a degree in marketing. His work experience consisted of serving as an apprentice carpenter for Burke Construction Company in Miami for approximately two months and as a contractor to build residential homes in Houston Texas. Milch voiced his opinion that the exam questions failed to satisfy the statutory requirements of being "objective" within the meaning of Chapter 468, Florida Statutes. Joseph Cole, the founder of Cole Construction College in 1949, testified that he had approximately 30 years experience in teaching construction, architectural and engineering courses. He had conducted various seminars for students and received a B.S. Degree from the University of Miami. He received a B.S. Degree in biochemistry and civil engineering from the University of Pittsburgh and conducted seminars at the University of Florida in Math, Physics and Engineering. He also conducted seminars at the Markowitz Engineering School. He was licensed in 1947 in Coral Gables, Miami Beach, and in Miami where he has built approximately 2,000 single-family homes, high-rise buildings and apartments. He aired his opinion that Milch missed approximately 36 questions of which approximately 24 were what he regarded as "impossible" questions. He expressed his awareness that during the morning sessions two questions were voided and credits were given to all examinees having a point value of 1 point each. Two questions were also voided from the afternoon session. Thomas H. Hebert, an associate of the testing agency which compiled the exam for the Board, i.e., Bryon, Harlow, Schaefer, Reed and Associates, explained the procedure for compiling the tests for the Board (Respondent). He stated that data is taken from Board references and an exam format is established. Examinees are tested on plan reading and estimating using standard plans for takeoff and specification requirements. The test is first administered to contractors and others who had previously passed the exam. This is done to test time limits, etc., and grading procedures. By so doing, he testified that it is possible to correct deficiencies in the exam. After the examination is compiled and is administered to the agency employees and other contractors who have passed the exam, an item analysis is compiled and computerized. There are five possible answers for each question. The exam is divided into three segments, i.e., the upper 27 percent, the mid 46 percent, and the lower 27 percent. After the tests results are in, the weighting on various questions are checked to see if large numbers of examinees "jump" one question and further to see if questions are ambiguous. If found to be or that there are two correct answers for a given question, credit is given for both answers. Thereafter, a discrimination index is compiled based on the lower and upper 27 percent. These papers are scrutinized and if there is a discrepancy in excess of .1 to .8 percent, the question is examined and a solution is derived at based on results gathered from the scrutiny. He testified further that if an exam paper is mutilated or is otherwise difficult to machine score, it is hand graded. All exam papers in which the score ranged from 0 to 30 are hand graded as are those where the score ranges between 60 to 70. Of those questions where there is no correct answer, the question is deleted and a new base is established. For example, if a question is deemed faulty, each question has a weight of 1.1 one hundredths of a point. If they have three correct answers, points are given for all three answers. It was further brought out during his testimony that it was not necessary for a contractor to pass the certification examination in order to practice contracting in Florida. Evidence reveals that there are two kinds of licenses issued by the Board, i.e., registration and certification. The registration process only requires compliance with local requirements and the filing of a form with the Board, which may be the passing of a local competency exam or simply obtaining a local occupational license. The Certification method is optional and if the contractor passes the certification examination, it is unnecessary for him to take any local examinations. After going over various questions missed by Petitioner Dennis Milch, Petitioners argue that the scope of the Board Certification Examination included questions affecting the business of contracting as well as the technical aspects of for example how to nail two boards together to make a safe structure. Florida Statutes Chapter 468.106(2)(a) provides for an examination covering knowledge of basic principles of contracting and construction. Chapter 468.101 declares the purpose of Chapter 468 and states in pertinent part that "any person desiring to obtain a certificate to engage in the business shall be required to establish his competency and qualifications." Hence, the legislature has covered the business of contracting as well as the theory of construction. This serves the purpose of Chapter 468 by making it safer for owners to contract with the contractors and to have assurances that no liens will be placed on their property by subcontractors, that the owners are safe from suit from work that was done on the job, that the payments made on the construction will not be diverted and that the contractor understands his obligations. This requires general knowledge of the mechanics lien law, basic contract law and workers compensation law, all of which were tested by the subject examination. Respecting Petitioners' argument that they were denied certain constitutional guarantees when they were instructed by Respondent to select the best possible answer but that after the test was administered and Respondent determined that many questions had no best choice, the Board failed to delete such questions from the exam, it was noted that after the Respondent discovered that several test questions were deemed acceptable but that the answers offered did not meet the tests of selecting the best possible answer, adjustments were made. In other words, there was no single best possible answer for approximately four questions. Rather than deleting the entire question, Respondent permitted those examinees who selected either the answer originally preferred by the Board or one of the later adopted alternate answers to achieve full credit for such questions and answers. The statute (Chapter 468, F.S.) mandates that the examination be an "objective" written examination. The criteria of objectivity is not met where the examining body is granted the discretion to accept alternate answers to a given question. A "best" answer is something different from an acceptable answer. To give the Board discretion to accept alternate answers would authorize a substitution of standards which is' not permitted by Chapter 468, F.S. Once subjectivity comes into play, Respondent becomes vested with almost unbridled discretion in deciding who shall become a certified general contractor. This was prohibited by the legislature by requiring objectivity in setting a uniform minimum test grade. As relates to Petitioner Milch, it was noted that a subsequent review of his exam resulted in his being awarded a half credit for his answer to question number 39 and the Board determined that after review answers B and A were both correct. The net result of this was that his overall score was 68. A review of the court cases revealed that Florida courts have not been involved in the minute details of how examination grades or points are awarded. See the cases of State ex rel. Topp v. Board of Electrical Examiners, 101 So.2d 583 (Fla. App. 1st 1968), and State ex real. Lane v. Dade County 258 So.2d (347 Fla. App. 3rd, 1972). These cases generally show that unless there is a clear abuse of discretion, courts shall not substitute their judgment for the agencies as to how examinations are graded. Petitioners also submit that once regrading had commenced, the Board should have deleted all questions with wrong answers or more than one equally acceptable answer, distributed the weight of the deleted questions in proportionate fashion of the remaining questions and considered passing to be 70 percent of the total points available, rather than 70 cumulative points. During the hearing, Petitioners failed to show how they were injured by the difference in the award of the points for questions deleted. A wrong without damage does not constitute a good cause of action. Based on the evidence presented it appears that the Petitioners are treated the same as all other examinees. Since the Petitioners have failed to establish that if the assignment of points were different, they would have passed the examination, this argument is moot. Petitioners also alleges that they were denied certain constitutional protections by Respondent's failure to adopt and promulgate uniform rules and regulations concerning preparation, administration and review of licensing examinations. Florida Statutes, Chapter 468, requires Respondent to conduct its affairs pursuant to Florida Statutes Chapter 120. The Administrative Procedure Acts set out specific procedures to be followed by State agencies in adopting, promulgating and enforcing rules. Statutory authority governing the granting of a license should be strictly followed. In this case, there is no evidence of the existence of any unlawful rule or regulation adopted by Respondent to govern any of the variety of issues concerning the licensing of general contractors. Petitioners also submit that the Board should be required to promulgate and enforce rules concerning examinations and appeals or results thereof and cite the reasons for the actions it takes prior to its review of the examination. The Board is not required to adopt rules and regulations in every area in which it is authorized to act by statute. Where the statute is clear, there is no requirement or reason by the Board to adopt rules. Here, the Board provided the applicants with a chance to examine the questions, their papers, grades and to complain if they wished about the questions either individually or at board meetings with the possibility that the fairness of the questions could be resolved quickly and informally and if necessary, as in this case, without the full ponoply of an administrative hearing. By so doing, the Board was clearly following its statutory duty to provide the applicants with a chance to see their examination papers and grades. (F.S. Chapter 466.110). Based on the above, it is concluded that Respondent compiled the May 23, 1975 examination based on objective standards. When the Board determined that certain questions were defective either because there was more than one answer or for other reasons, the Board reviewed said questions and credited those examinees who failed to properly answer the question. By so doing, Petitioners were treated the same as all examinees who took the exam. Based on the record evidence, it further appears that all the questions meet the statutory tests of being objective and the Board's determination that a cumulative score of 70 percent is necessary to successfully obtain a certification, was not shown by any competent or substantial evidence to be an abuse of discretion. It is therefore recommended that the agency's action be sustained and that the petition filed herein be dismissed.

Recommendation Based on the foregoing findings of fact and conclusions of law, it is therefore recommended that the agency's actions be affirmed and the petition filed herein be DISMISSED. DONE and ENTERED this 17th day of January, 1977, in Tallahassee, Florida. JAMES E. BRADWELL, Hearing Officer Division of Administrative Hearings Room 530, Carlton Building Tallahassee, Florida 32304 (904) 488-9675

Florida Laws (1) 119.07
# 9
ERIC SOBEL vs DEPARTMENT OF BUSINESS AND PROFESSIONAL REGULATION, BOARD OF CONSTRUCTION, 03-001642 (2003)
Division of Administrative Hearings, Florida Filed:Clearwater, Florida May 07, 2003 Number: 03-001642 Latest Update: Nov. 06, 2019

The Issue The issues in this case are whether certain questions within the June 2002 construction building contractor examination are invalid, and whether Petitioner should receive credit for certain answers scored as incorrect.

Findings Of Fact In June 2002, Petitioner sat for the construction building contractor examination. Shortly following the exam, Petitioner was advised that he incorrectly answered 17 of the 50 exam questions and did not attain the minimum passing score of 70 percent, but received a failing scaled score of 66 percent. Petitioner timely challenged the validity and scoring of eight questions, including questions 8, 14, 17, 33, 34, 38, 43, and 44. In order for Petitioner to acquire a passing score, Petitioner must prove that certain challenged questions are invalid or demonstrate that he is entitled to receive credit for his answers. Specifically, Petitioner must demonstrate that either three questions should be stricken from the exam providing Petitioner with 70.2 percent, two questions should be stricken and one answer scored as correct providing Petitioner with 70.8 percent or two answers should scored as correct providing Petitioner with 70 percent. QUESTION 8 Exam Question 8 asks, "According to AIA-A201, who determines the interest rate that the contractor can charge on due and unpaid payments?" Petitioner's expert, Mr. Uman, argues that the parties to the contract are not defined within the question and it is therefore misleading. However, the credited answer D, "all the parties must agree on the rate" is within the provided reference material and is clearly the best answer. It is not misleading and Petitioner's argument lacks merit. In addition, 89.47 percent of the test-takers correctly answered Question 8. QUESTION 14 Exam Question 14 is wordy and involves computations. It requires the test-taker to calculate the number of "labor" hours required per 100 pieces to build a wall, given certain pricing and wall construction information. Question 14 is ambiguous and confusing on its face. While the question asks for labor hours, the facts provide a fixed combined hourly cost for a mason and laborer's hour. There is no distinction made between "labor" hours and a "laborer's" hours. Mr. Collier admitted that there is some apparent confusion between "labor" costs and the "laborer's" costs. Mr. Palm further agreed and indicated that he fully understood Petitioner's rationale to divide the labor costs in half and choose answer A. Furthermore, it is clear that Petitioner's perception of the question was not unique. In fact, only 46.62 percent of the test-takers correctly answered Question 14. QUESTION 17 Exam Question 17 asks, "During the bid process, which document has priority in the event of conflicting information?" Clearly, the correct answer is B, "addenda." Petitioner's argument regarding "competitively bid projects" is without merit. Mr. Palm succinctly explained that Petitioner's selection was obviously incorrect because "plans don't change during the bid process unless there is an addenda issued." Moreover, 75.56 percent of the test-takers correctly answered Question 17. QUESTION 33 Exam Question 33 identifies a situation that where drawings differ from written specifications and where there is no legal precedent that one is more binding than the other. The question specifically calls for the best procedure according to the listed and available reference. While Mr. Uman argues that the answer does not appear within the reference material in a clear manner, the exact text of the question and answer are in fact within the material. Petitioner's argument lacks credibility. QUESTION 34 Exam Question 34 asks the test-taker "what is the EARLIEST workday for completing the masonry work?" given the number of crew, the number of hours required, and the ratio constant of the crew. Although 80.45 percent of the test-takers correctly answered Question 34, Mr. Uman argues that the question could have been answered without reference to the Builder's Guide to Accounting material and therefore, was misleading. Petitioner's argument is devoid of common sense. QUESTION 38 Exam Question 38 asks the test-taker to identify the activity that "a specialty structural contractor is qualified" to perform. Petitioner's expert, Mr. Uman, again argues that the question is misleading since the credited correct answer "perform non-structural work" is not written verbatim in the provided reference material. To the contrary however, all of the alternative choices are clearly listed in the reference material as activities specifically prohibited by specialty structure contractors. Furthermore, page 2B17 to 61G415.015 of the Contractor's Manual specifically states that: The specialty structure contractor whose services are limited shall not perform any work that alters the structural integrity of the building including but not limited to roof trusses. Respondent's experts, Mr. Collier and Mr. Palm, agree that Question 38 is clear. Moreover, 53.38 percent of test- takers correctly answered the question. While the question appears to require enhanced reasoning skills and is generally more difficult, it is not misleading. Petitioner's assertions are without merit. QUESTION 43 Exam Question 43 asks, "Which accounting method should be used by a contractor if the contractor is unable to reasonably estimate the amount of progress to date on a job or the total costs remaining to fulfill the contract?" Mr. Uman argues that the question is ambiguous and the reference material is "not terribly clear." He further alleges that when a contractor cannot estimate progress, the contractor cannot establish a "completed contract method," the credited correct answer. Respondent's experts disagree. While it is true that Mr. Palm agreed that all of the choices are accounting methods which is inconsistent with Mr. Collier's testimony, the reference material is clear. In fact, 58.65 percent of the test-takers correctly answered Question 43. Petitioner presented insufficient evidence that he should receive credit for his answer or that Question 43 should be invalidated. QUESTION 44 Exam Question 44 provides detailed information regarding a standard form contract and asks, "Based ONLY on the information given above, what is the amount of the current payment due?" In addition, however, as Mr. Uman points out, the standard form referred to in the problem was mistakenly misidentified as Form 201 instead of Form 702. While it is clear that the referenced form was mislabeled, the precise form number was incidental, unrelated to the question, and unnecessary to compute the answer. In fact, Mr. Palm explains that the problem was "just a mathematical exercise." According to Mr. Collier, the question was not misleading, and the incorrect reference was irrelevant. "It's simple math, industry knowledge." Furthermore, Petitioner's answer is clearly incorrect because "he failed to deduct the retainage." Finally, 54.89 percent of the test-takers correctly answered Question 44.

Recommendation Based on the foregoing Findings of Fact and Conclusions of Law, it is RECOMMENDED that a final order be entered invalidating only Question 14, re-computing Petitioner's examination score, and dismissing his challenge. DONE AND ENTERED this 1st day of October, 2003, in Tallahassee, Leon County, Florida. S WILLIAM R. PFEIFFER Administrative Law Judge Division of Administrative Hearings The DeSoto Building 1230 Apalachee Parkway Tallahassee, Florida 32399-3060 (850) 488-9675 SUNCOM 278-9675 Fax Filing (850) 921-6847 www.doah.state.fl.us Filed with the Clerk of the Division of Administrative Hearings this 1st day of October, 2003. COPIES FURNISHED: Nickolas Ekonomides, Esquire 791 Bayway Boulevard Clearwater, Florida 33767 Charles F. Tunnicliff, Esquire Department of Business and Professional Regulation 1940 North Monroe Street, Suite 60 Tallahassee, Florida 32399-2202 Nancy P. Campiglia, General Counsel Department of Business and Professional Regulation Northwood Centre 1940 North Monroe Street Tallahassee, Florida 32399-2202 Robert Crabill, Executive Director Construction Industry Licensing Board Department of Business and Professional Regulation Northwood Centre 1940 North Monroe Street Tallahassee, Florida 32399-0792

Florida Laws (3) 120.57120.68455.217
# 10

Can't find what you're looking for?

Post a free question on our public forum.
Ask a Question
Search for lawyers by practice areas.
Find a Lawyer